Разное

Загадки о цифрах: Загадки про цифры с ответами

Содержание

Загадки про цифры и числа для детей с ответами

Смотрите загадки с ответами: два, четыре, ноль, три, пять, шесть, семь, восемь, девять, один.

Ах, какая запятая
На листе лежит большая!
Можно ею даже мерить,
Просто это цифра…
Ответ: девять
12017

Друг на друге два кольца,
Удалых два молодца.
В числовой их ряд попросим
И получим цифру…
Ответ: восемь
Ларионов Юрий, Москва
4001

Она похожа на топор,
Но не колит дров во двор.
Как коса, но не совсем,
Просто это цифра…
Ответ: семь
7835

Я устроила парад,
Цифры строю, как солдат!
И порядок четкий есть —
После пять шагает…
Ответ: шесть
6490

В школе надо не лениться:
Рисовать, писать, учиться,
На уроках отвечать
И в дневник поставят…
Ответ: пять
6994

Решили фильм про цифры снять,
Стали камеры включать.
Откуда буква Ч в эфире?

Не бойтесь! Цифра я!…
Ответ: четыре
5754

Букву З я обведу,
К цифрам в гости приведу.
Ты внимательней смотри —
Получилась цифра…
Ответ: три
7702

Светит солнце, пруд цветет,
Лебедь по нему плывет,
Ближе он подплыл едва —
Оказалась цифра…
Ответ: два
6122

Не забьешь так просто гол,
На воротах стоит кол.
И в бою с ним не сразиться,
Это цифра…
Ответ: единица
6672

Встали цифры, как отряд,
В дружный числовой свой ряд.
Первой по порядку роль
Нам сыграет цифра…
Ответ: ноль
5909

Кто-то ночью старый стул
Спинкой вниз перевернул.
И теперь у нас в квартире
Стал он цифрою…
Ответ: четыре
5808

Лебедь плавает в тетрадке,
Значит что-то не в порядке.
Если ты совсем Незнайка,
Цифру эту получай-ка.
Ответ: два

5342

Вот задачка, отгадайте!
К шестёрке тройку прибавляйте.
Получается она —
Кружок и форма завитка.
Ответ: девять
5386

Два кружка соединилось,
Что за цифра получилась?
Два умножим на четыре —
И она уже в эфире.
Ответ: восемь
4477

Палочка, но с завитком,
Как кочерга — такая же на вид.
Эта цифра первая во всём,
И в счёте впереди других стоит!
Ответ: один
3935

Сколько в яркой радуге цветов?
Сколько на земле есть чудес света?
Сколько у Москвы всего холмов?
Нам цифра эта так подходит для ответа!
Ответ: семь
3820

А с этой цифрой лишь сплошные суеверия,
И так обидно: ну, за что ей это всё?
И место в счёте нам внушает лишь доверие:
Между пятёркой и семёркою оно.
Ответ: шесть
3076

У отличника повсюду
Эту цифру в дневнике найти.
Ещё одну подсказку не забуду —

Перед ней стоит шестёрка на пути.
Ответ: пять
3147

Как поётся в песенке одной:
Дважды два — такая будет цифра.
Перед пятёркой юбилейной, золотой,
Стоит она… Ну, кто ответит быстро?
Ответ: четыре
2660

На букву О она похожа, кругла собой.
Может в паре постоять с циферкой любой.
Не имеет ни начала, ни конца,
Носит гордое название нуля.
Ответ: ноль
2455

Сколько раз в дверь нужно постучать?
Сколько будет двойка плюс один?
Поскорее нужно это отгадать,
Эта цифра с буквой З — один в один.
Ответ: три
2455

Если эту цифру в школе получить,
То родители вас дома наругают.
С лебедем её легко сравнить,
Про цифру эту все детишки знают?
Ответ: два
2360

Эта цифра так похожа
На красивый парус!
Раздувается всё шире
Циферка..
Ответ: четыре

3259

Цифра эта так хитра,
Ты её переверни,
И начнутся чудеса —
Цифру шесть увидишь ты!
Ответ: девять
4118

Два кружочка рядом встали —
Что за дивные очки?
Поверни хоть вверх ногами,
Одинаковы они!
Ответ: восемь
3400

Что за цапля, каждый знает,
За шестёркою ступает.
Да, она известна всем
Эта цапля — цифра…
Ответ: семь
3295

Циферка, как бегемотик,
Круглый у неё животик,
Шейку ловко изгибает,
За пятёркою ступает.
Ответ: шесть
2808

Циферка на крюк похожа,
Что на автокране.
И на двоечку похожа,
Если вверх ногами.
Ответ: пять
2444

Необычна цифра эта,
Круглолица, так и знай.
И на букву З похожа,
Что за цифра? Отгадай!
Ответ: три
2415

Гибко шейку изгибает,
И красива, и стройна,
Ловко хвостик поднимает,

Что за цифра? Цифра…
Ответ: два
2465

На стебелёк она похожа,
Стоит почтенно, как вельможа.
Прямая, ровная всегда,
После нуля идёт она.
Ответ: один
2256

Эта цифра так похожа
На букву алфавита О.
Но без цифорок других
Она не значит ничего.
Ответ: ноль
2204

Проживают в трудной книжке
Хитроумные братишки.
Десять их, но братья эти
Сосчитают всё на свете.
Ответ: Цифры
483



Смотрите загадки на другие темы:


Загадки с цифрами – Математические загадки

Цифра «1»

Стоит Антошка на одной ножке; его ищут, а он не откликается (гриб).

На ноге стоит одной, крутит-вертит головой.

Нам показывает страны, реки, горы, океаны (глобус).

На длинной ножке, застыв до поры, отдыхает палочка после игры (единица).

У кого одна нога, да и та без башмака? (гриб).

Цифра «2»

Два братца пошли в реку купаться (вёдра).

 Два кольца, два конца, посередине – гвоздик (ножницы).

Два домика – теплушки подарены Танюшке (варежки).

Шея длинная такая, хвост крючком…

И не секрет, любит всех она лентяев, а ее лентяи – нет! (двойка).

Есть совсем другая птица: если сядет на страницу, то с поникшей головой,   возвращаюсь я домой (двойка).

Цифра «3»

Есть спина, а не лежит никогда. Есть четыре ноги,  а не ходят и три. Сам всегда стоит, а всем сидеть велит (стул).

Я стою на трех ногах, ноги в черных сапогах. Зубы белые, педаль. Как зовут меня? (рояль).

Входишь в одну дверь, а выходишь из трех. Думаешь, что вышел, а на самом деле – вошел (рубашка).

Треугольная доска, а на ней три волоска. Волосок – тонкий, голосок – звонкий (балалайка).

Три братца пошли на реку купаться.

Два купаются, третий на берегу валяется. Искупались – вышли, на третьем повисли (ведра и коромысло).

Трое одним лугом пашут (пальцы пишут).

У него глаза цветные, не глаза, а три огня, он по очереди  ими сверху смотрит на меня (светофор).

Цифра «4»

 

Четыре братца под одной крышей стоят (стол).

На 4 ногах стою, ходить я вовсе не могу. Когда устанешь ты гулять, ты можешь сесть и отдыхать (стул).

На 4 ноги надевали сапоги. Перед тем как надевать, стали обувь надувать (шины).

 Шевелились у цветка все четыре лепестка. Я сорвать его хотел, он вспорхнул и улетел (бабочка).

Ежегодно приходят к нам в гости: один седой, другой молодой, третий скачет, а четвертый плачет (времена года).

Цифра «5»

Есть, друзья, такая птица: если сядет на страницу, очень рад бываю я, а со мною вся семья (пятерка).

Пятерка братьев неразлучна, им вместе никогда не скучно. Они работают пером, пилою, ложкой, топором (пальцы).

У пяти братьев одна работа (пальцы).

У двух матерей по пяти сыновей, одно имя всем (пальцы).

5 мешочков шерстяных – греются братишки в них (перчатки).

Цифра «6»

Если на голову встанет, ровно на три больше станет (шесть).

На дворе переполох, с неба сыплется горох. Съела 6 горошин Нина, у нее теперь ангина (град).

6  ног, 2 головы, один хвост. Кто это? (всадник на коне).

Цифра «7»

Ежедневно в 7 утра, я трещу: вставать порррррра! (будильник).

Есть 7 братьев: годами равные, именами разные (дни недели).

Братьев этих ровно 7. Вам они известны всем. Каждую неделю кругом ходят братья друг за другом. Попрощается последний – появляется передний (дни недели).

Пять щенят, да мама-лайка. Ты попробуй, сосчитай-ка! (6)

Цифра «8»

Я так мила, я так кругла, я состою из двух кружочков. Как я рада, что нашла себе таких, как вы дружочков (8).

Цифра «9»

Отгадайте-ка, ребятки, что за цифра акробатка? Если на голову встанет, ровно на три меньше станет (9).

Цифра «10»

Твои помощники – взгляни – десяток дружных братцев.

Как славно жить, когда они работы не боятся (пальцы).

Проживают в умной книжке хитроумные братишки. 10 их, но  братья эти сосчитают все на свете (цифры).

Есть у меня работники, во всем помочь охотники. Живут не за стеной – день и ночь со мной: целый десяток, верных ребяток! (пальцы).

Загадки на числа

70 одёжек, а все без застёжек  (капуста).

Сели на страничке 33 сестрички. Сели рядом – не молчат, нам загадки говорят (буквы).

Кулик – не велик, целой сотне велит: то сядь, да учись; то встань, разойдись (школьный звонок).

У меня знакомых – тьма. Не могу их счесть сама, потому что кто пройдет, тот и руку мне пожмет (дверь).

Сотнями глаз во все стороны глядит (наперсток).

В два ряда дома стоят – 10, 20, 100 подряд. И квадратными глазами друг на друга глядят (улица).

12 братьев равно называются и разными делами занимаются (месяцы года).

Рассыпался горох на семьдесят дорог: никто его не подберет (град).

Золотист он и усат. В ста карманах – 100 ребят (колос).

Лето целое старалась – одевалась, одевалась…  А как осень подошла, нам одёжки отдала. Сотню одежонок сложили мы в бочонок (капуста).

Тысяча братьев одним поясом подпоясаны (колосья в снопу).

Один пастух 1000 овец пасет (месяц и звезды).

Золотое решето черных домиков полно.  Сколько чёрненьких домков, столько беленьких жильцов (подсолнух).

 Сто березовых солдат, взявшись за руки, стоят. Днем и ночью, круглый год: охраняют огород (забор).

Обучающие загадки про цифры с ответами

Сколько лет в яйце цыпленку,
Сколько крыльев у котенка,
Сколько в алфавите цифр,
Сколько гор проглотит тигр,
Сколько мышка весит тонн,
Сколько в стае рыб ворон,
Сколько зайцев съела моль,
Знает только цифра… (ноль)

Мячик Скачет по страницам.
Ищет он свою сестрицу,
Что имеет вид кольца –
Без начала и конца.

Он похож на колобок,
Он пузат и круглобок.
На него похожа Кошка,
Если сложится в клубок.

Сколько солнышек за тучкой,
Сколько стержней в авторучке,
Сколько у слона носов,
Сколько на руке часов?
Сколько ног у мухомора
И попыток у сапера,
Знает и гордится собой,
Цифра-столбик… (единица)

Стоит она среди листа
Одна, когда тетрадь пуста.
Задрав свой нос до потолка,
Она бранит ученика.
И словно цапля средь болот
Его за лень его клюет.
Хоть у нее одна нога
Она стройна, горда, строга.

Ни журавль то, ни синица.
А всего лишь…(единица)

С хитрым носиком сестрица
Счет откроет …(единица)

Сотня лун бы появилась –
И тогда бы осветилась
Ночь как день! Но, жаль, луна
Светит нам всегда …(одна)

Сколько ушек на макушке,
Сколько ног у пол-лягушки,
Сколько у сома усов
У полюсов планеты,
Сколько в целом половинок,
В паре – новеньких ботинок,
И передних лап у льва
Знает только цифра… (два)

Людям всем дано от Бога
По одной лишь голове!
Ну, а что руки, а ноги?
Их у каждого по …(две)

Что скользит по светлой глади
Ученической тетради
Белым лебедем прекрасным,
Ставшим от позора красным
За бездельника, плутишку
Непослушного мальчишку?
То, за что его ругают
И конфет в обед лишают.

С легким росчерком пера
Появилась цифра…(два)

Лебедь плавает в тетрадке,
Значит что-то не в порядке.
Если ты совсем Незнайка,
Эту Цифру получай-ка.

Сколько месяцев в зиме,
В лете, в осени, в весне,
Сколько глаз у светофора,
Баз на поле для бейсбола,
Граней у спортивной шпаги
И полос на нашем флаге,
Что нам кто ни говори,
Знает правду цифра… (три)

Вот так чудо! Ну-ка, ну-ка,
Ты получше посмотри –
Это вроде бы и буква,
Но еще и цифра …(три)

Эту Цифру угадай-ка!
Она большая зазнавай-ка.
Единицу сложишь с двойкой,
И получишь цифру …(три)

Эта цифра просто чудо.
У нее родня повсюду.
Даже в алфавите есть
У нее сестра-близнец.

Сколько лапок у мангуста,
Лепестков в цветке капусты,
Пальцев на куриной ножке
И на задней лапе кошки,
Рук у Тани вместе с Петей
И сторон всего на свете
Да и океанов в мире,
Знает циферка… (четыре)

Я у бабушки была, –
У нее во всей квартире
Три огромные стола,
Ног у каждого – …(четыре)

Кто-то ночью старый стул
Спинкой вниз перевернул.
И теперь у нас в квартире
Стал он цифрою …(четыре)

То ли цифра, то ли вилка,
То ли развилка двух дорог.
В ученической тетради
Знаю точно – все ей рады.

Сколько пальцев на руке
И копеек в пятачке,
У морской звезды лучей,
Клювов у пяти грачей,
Лопастей у листьев клена
И углов у бастиона,
Про все это рассказать
Нам поможет цифра… (пять)

На руке малышка Лена
Любит считать пальчики!
У нее, на удивленье,
Каждый раз выходит …(пять)

Смотрит мама с нетерпеньем
На страницы дневника.
Ждет заветную оценку
У сынка-озорника.
Но опять одни четверки.
Нет красавицы…(пятерки)

Если ДВА перевернуть
И внимательно взглянуть,
Так и сяк взглянуть опять,
То получим цифру …(пять)

Сколько букв есть у дракона
И нулей у миллиона,
Разных шахматных фигур,
Крыльев у трех белых кур,
Ног у майского жука
И сторон у сундука.
Коль не можем сами счесть,
Нам подскажет цифра… (шесть)

А моей подруге Тоне
Довелось на пони сесть,
И у Тони вместе с пони
Стало ножек сразу …(шесть)

Глядела цифра в зеркало
И о сестре мечтала.
Но только одного свойства
Его видать не знала.
И получила двойника.
Как капелька водицы
Сестра похожа на нее.
Да только вниз косица. (шесть и девять)

Если навесной замок
Вверх поднимет хоботок,
То тогда увидим здесь
Не замок, а цифру …(шесть)

Сколько в радуге цветов,
Дней в неделе у китов.
Гномиков у Белоснежки,
Братьев-близнецов у пешки,
Нот, что знают даже дети,
И всего чудес на свете,
Разобраться с этим всем
Нам поможет цифра… (семь)

Есть пословица у нас
И она известна всем –
Отрезаешь только раз,
А отмерить надо …(семь)

Я – цифра, меньше десяти.
Меня легко найти.
Но если букве «я» прикажешь встать рядом,
Я все: отец, и ты, и дедушка, и мать…

Вьется по ветру коса,
А по спинке полоса

Сколько на море ветров,
И копыт у двух ослов,
Щупалец у осьминога,
И клыков у пары догов?
Сколько ног у паука,
Паука-крестовика?
Если мы про это спросим
Нам ответит цифра… (восемь)

– Сколько ты имеешь ножек? –
Осьминога тихо спросим.
Тот ответить нам не сможет,
Но и так мы знаем – …(восемь)

Цифра с виду как игрушка –
Неваляшка-погремушка.
Не удариться ей о земь.
Всем понятно – это…(восемь)

Сколько в дюжине пиратов,
Если три ушли куда-то,
Месяцев в году без лета,
Исполнителей нонета,
Жизней у бродячей кошки,
И в десятке мух без мошки?
Не ищи ответ нигде, ведь
Есть ответ у цифры… (девять)

Вот так циферка, взгляни,
У нее и есть фокус –
Ты переверни ее,
И получишь цифру шесть!

Нолик, стань за единицей,
За своей родной сестрицей.
Только так, когда вы вместе,
Называть вас будут …(десять)

 

ять мальчишек было в классе,
В класс вошел еще и Вася,
А потом Илья и Миша.
Сколько же теперь мальчишек?
(Ответ: Восемь)

Цифра с виду как игрушка —
Неваляшка—погремушка.
Не удариться ей о земь.
Всем понятно — это…
(Ответ: Восемь)

С легким росчерком пера
Появилась цифра…
(Ответ: Два)

У Сережи карандашик
И еще один — у Даши.
Сколько же у малышей
На двоих карандашей?
(Ответ: Два)

Что скользит по светлой глади
Ученической тетради
Белым лебедем прекрасным,
Ставшим от позора красным
За бездельника, плутишку
Непослушного мальчишку?
То, за что его ругают
И конфет в обед лишают.
С легким росчерком пера
Появилась цифра…
(Ответ: Два)

Я к реке иду из дома,
А навстречу — сто знакомых.
Вдруг один из ста ребят,
Тот, что был с сачком в руке,
Повернуть решил назад.
Сколько нас идет к реке?
(Ответ: Двое)

У одного человека было 17 овец.
Все, за исключением 9, разбежались.
Сколько овец осталось?
(Ответ: Девять)

Как безлиственная ветка,
Я пряма, суха, тонка.
Ты встречал меня нередко
В дневнике ученика.
(Ответ: Единица)

Стоит она среди листа
Одна, когда тетрадь пуста.
Задрав свой нос до потолка,
Она бранит ученика.
И словно цапля средь болот
Его за лень его клюет.
Хоть у нее одна нога
Она стройна, горда, строга.
Ни журавль то, ни синица.
А всего лишь…
(Ответ: Единица)

Скачет мячик по страницам.
Ищет он свою сестрицу,
Что имеет вид кольца —
Без начала и конца.
(Ответ: Ноль)

Сколько дней рождения у человека,
Дожившего до 80 лет?
(Ответ: Один)

У меня есть две игрушки,
Завтра дам одну Ванюшке.
День рождения у Вани,
Отнесу ему коня.
Сколько же игрушек станет
Завтра дома у меня?
(Ответ: Одна)

Смотрит мама с нетерпеньем
На страницы дневника.
Ждет заветную оценку
У сынка—озорника.
Но опять одни четверки.
Нет красавицы…
(Ответ: Пятерки)

В нашей группе есть Антоша,
Нина, Коля и Алеша,
Витя, Ира, Вова, Маша,
Соня, Кира и Наташа,
Две Марины, Света, Миша.
Сколько же всего детишек?
(Ответ: Пятнадцать)

Горело 5 электрических лампочек,
3 лампочки выключили.
Сколько лампочек осталось?
(Ответ: Пять)

За окном сидели птички,
Голубь, дрозд и три синички.
Спросим мы учеников
И прилежных учениц:
«Кто ответить нам готов,
Сколько за окошком птиц?»
(Ответ: Пять)

Вьется по ветру коса,
А средь спинки полоса.
(Ответ: Семь)

Сестры—белочки сидели
Вшестером в дупле на ели.
К ним еще одна примчалась —
От дождя она спасалась.
Все теперь сидят в тепле.
Сколько белочек в дупле?
(Ответ: Семь)

Я — цифра, меньше десяти.
Меня легко найти.
Но если букве «я» прикажешь рядом встать,
Я всё: отец, и ты, и дедушка, и мать…
(Ответ: Семь)

Летели утки: одна впереди и две позади,
Одна позади и две впереди,
Одна между двумя.
Сколько их было всего?
(Ответ: Три)

Столько книжек у ребяток,
Сколько у Алеши пяток.
Принесла ребяткам Галя
Мячик, книжку, мишек.
Вы, ребята, посчитали,
Сколько стало книжек?
(Ответ: Три)

У меня есть две конфетки,
Дам одну сестренке Светке.
Я не жадный, и за это
Дал мне папа две конфеты!
И теперь конфеток стало,
Даже больше, чем сначала!
(Ответ: Три)

Эта цифра просто чудо.
У нее родня повсюду.
Даже в алфавите есть
У нее сестра—близнец.
(Ответ: Тройка)

Проживают в умной книжке
Хитроумные братишки.
Десять их, но братья эти
Сосчитают все на свете.
(Ответ: Цифры)

То ли цифра, то ли вилка,
То ли двух дорог развилка.
В ученической тетради
Знаю точно — все ей рады.
(Ответ: Четверка)

К трем лягушкам у болота
Прибежали два енота,
Прискакала тетя жаба
И пришла наседка Ряба.
Сколько в камышах болотных
Оказалось земноводных?
(Ответ: Четверо)

Чтоб одеть тепло сыночков,
Не хватает двух носочков.
Сколько же в семье сынков,
Если в доме шесть носков?
(Ответ: Четверо)

Света принесла из школы
Три пятерки за глаголы,
За предлоги — тройку,
А за суффикс — двойку.
Сколько в дневнике у Светы
Положительных отметок?
(Ответ: Четыре)

У броненосца две передние ноги,
Две задние, да еще две левые
И две правые.
Сколько всего ног?
(Ответ: Четыре)

Шел охотник в лес.
Встретил двух охотников
Да одного грибника.
Сколько всего человек шло в лес?
(Ответ: Четыре)

Я сестру свою баюкал,
Утешал я Ксюшу,
Положил я к ней трех кукол,
Зайчика из плюша.
Сколько же всего игрушек
У моей сестренки Ксюши?
(Ответ: Четыре)

Сколько концов у трех палок?
(Ответ: Шесть)

У Надюши пять тетрадок,
Кляксы в них и беспорядок.
Нужен Наде черновик.
Вася, первый ученик,
Дал еще тетрадку Наде
Сколько у нее тетрадей?
(Ответ: Шесть)

Глядела цифра в зеркало
И о сестре мечтала.
Но только свойства одного
Его видать не знала.
И получила двойника.
Как капелька водицы
Сестра похожа на нее.
Да только вниз косица.
(Ответ: Шесть и девять)

Проживают в умной книжке
Хитроумные братишки.
Десять их, но братья эти
Сосчитают все на свете.
(Цифры)

Встали цифры, как отряд,
В дружный числовой свой ряд.
Первой по порядку роль
Нам сыграет цифра… (0)

Он похож на колобок,
Он пузат и круглобок.
На него похожа Кошка,
Если сложится в клубок.
(0)

Скачет мячик по страницам.
Ищет он свою сестрицу,
Что имеет вид кольца —
Без начала и конца.
(0)

Сколько лет в яйце цыпленку,
Сколько крыльев у котенка,
Сколько в алфавите цифр,
Сколько гор проглотит тигр,
Сколько мышка весит тонн,
Сколько в стае рыб ворон,
Сколько зайцев съела моль,
Знает только цифра…
(Нуль)

Эта цифра так похожа
На букву алфавита О.
Но без циферок других
Она не значит ничего.
(0)

Как безлиственная ветка,
Я пряма, суха, тонка.
Ты встречал меня нередко
В дневнике ученика.
(Единица)

На стебелёк она похожа,
Стоит почтенно, как вельможа.
Прямая, ровная всегда,
После нуля идёт она.
(1)

Не забьешь так просто гол,
На воротах стоит кол.
И в бою с ним не сразиться,
Это цифра… (Единица)

Сколько солнышек за тучкой,
Сколько стержней в авторучке,
Сколько у слона носов,
Сколько на руке часов?
Сколько ног у мухомора
И попыток у сапера,
Знает и собой гордится,
Цифра-столбик…
(Единица)

Стоит она среди листа.
Одна, когда тетрадь пуста.
Задрав свой нос до потолка,
Она бранит ученика.
И словно цапля средь болот
Его за лень его клюет.
Хоть у нее одна нога
Она стройна, горда, строга.
Ни журавль то, ни синица.
А всего лишь…
(Единица)

С хитрым носиком сестрица
Счёт откроет…
(Единица)

Гибко шейку изгибает,
И красива, и стройна,
Ловко хвостик поднимает,
Что за цифра? Цифра… (2)

Если эту цифру в школе получить,
То родители вас дома наругают.
С лебедем её легко сравнить,
Про цифру эту все детишки знают?
(2)

Лебедь плавает в тетрадке,
Значит что-то не в порядке.
Если ты совсем Незнайка,
Цифру эту получай-ка.
(2)

Светит солнце, пруд цветет,
Лебедь по нему плывет,
Ближе он подплыл едва —
Оказалась цифра… (2)

Сколько ушек на макушке,
Сколько ног у пол-лягушки,
Сколько у сома усов
У планеты полюсов,
Сколько в целом половинок,
В паре — новеньких ботинок,
И передних лап у льва
Знает только цифра…
(2)

С легким росчерком пера
Появилась цифра… (2)

Что скользит по светлой глади
Ученической тетради
Белым лебедем прекрасным,
Ставшим от позора красным
За бездельника, плутишку
Непослушного мальчишку?
То, за что его ругают
И конфет в обед лишают.
С легким росчерком пера
Появилась цифра… (2)

Букву З я обведу,
К цифрам в гости приведу.
Ты внимательней смотри —
Получилась цифра… (3)

Вот так чудо! Ну-ка, ну-ка,
Ты получше посмотри –
Это вроде бы и буква,
Но еще и цифра… (3)

Необычна цифра эта,
Круглолица, так и знай.
И на букву З похожа,
Что за цифра? Отгадай!
(3)

Сколько месяцев в зиме,
В лете, в осени, в весне,
Сколько глаз у светофора,
Баз на поле для бейсбола,
Граней у спортивной шпаги
И полос на нашем флаге,
Что нам кто ни говори,
Знает правду цифра… (3)

Цифру эту угадай-ка!
Она большая зазнавай-ка.
Единицу сложишь с двойкой,
И получишь цифру … (3)

Эта цифра просто чудо.
У нее родня повсюду.
Даже в алфавите есть
У нее сестра—близнец.
(3)

Кто-то ночью старый стул
Спинкой вниз перевернул.
И теперь у нас в квартире
Стал он цифрою … (4)

Сколько лапок у мангуста,
Лепестков в цветке капусты,
Пальцев на куриной ножке
И на задней лапе кошки,
Рук у Тани вместе с Петей
И всего сторон на свете
Да и океанов в мире,
Знает циферка… (4)

То ли цифра, то ли вилка,
То ли двух дорог развилка.
В ученической тетради
Знаю точно — все ей рады.
(4)

Эта цифра так похожа
На красивый парус!
Раздувается всё шире
Циферка… (4)

В школе надо не лениться:
Рисовать, писать, учиться,
На уроках отвечать
И в дневник поставят… (5)

Если ДВА перевернуть
И внимательно взглянуть,
Так и сяк взглянуть опять,
То получим цифру… (5)

На руке малышка Лена
Любит пальчики считать!
У нее, на удивленье,
Каждый раз выходит… (5)

Сколько пальцев на руке
И копеек в пятачке,
У морской звезды лучей,
Клювов у пяти грачей,
Лопастей у листьев клена
И углов у бастиона,
Про все это рассказать
Нам поможет цифра… (5)

Смотрит мама с нетерпеньем
На страницы дневника.
Ждет заветную оценку
У сынка—озорника.
Но опять одни четверки.
Нет красавицы…
(Пятёрки)

Циферка на крюк похожа,
Что на автокране.
И на двоечку похожа,
Если вверх ногами.
(5)

А моей подруге Тоне
Довелось на пони сесть,
И у Тони вместе с пони
Стало ножек сразу… (6)

Если навесной замок
Вверх поднимет хоботок,
То тогда увидим здесь
Не замок, а цифру… (6)

Сколько букв есть у дракона
И нулей у миллиона,
Разных шахматных фигур,
Крыльев у трех белых кур,
Ног у майского жука
И сторон у сундука.
Коль не можем сами счесть,
Нам подскажет цифра… (6)

Циферка, как бегемотик,
Круглый у неё животик,
Шейку ловко изгибает,
За пятёркою ступает.
(6)

Я устроила парад,
Цифры строю, как солдат!
И порядок четкий есть —
После пять шагает… (6)

Вьется по ветру коса,
А средь спинки полоса.
(7)

Есть пословица у нас
И она известна всем –
Отрезаешь только раз,
А отмерить надо… (7)

На косу она похожа,
Но косить траву не может —
Не наточена совсем
И не косит цифра… (7)

Она похожа на топор,
Но не колит дров во двор.
Как коса, но не совсем,
Просто это цифра… (7)

Сколько в радуге цветов,
Дней в неделе у китов.
Гномиков у Белоснежки,
Братьев-близнецов у пешки,
Нот, что знают даже дети,
И всего чудес на свете,
Разобраться с этим всем
Нам поможет цифра… (7)

Что за цапля, каждый знает,
За шестёркою ступает.
Да, она известна всем
Эта цапля — цифра… (7)

Я — цифра, меньше десяти.
Меня легко найти.
Но если букве «я» прикажешь рядом встать,
Я всё: отец, и ты, и дедушка, и мать… (7)

Два кружочка рядом встали —
Что за дивные очки?
Поверни хоть вверх ногами,
Одинаковы они!
(8)

Друг на друге два кольца,
Удалых два молодца.
В числовой их ряд попросим
И получим цифру… (8)

Сколько на море ветров,
И копыт у двух ослов,
Щупалец у осьминога,
И клыков у пары догов?
Сколько ног у паука,
Паука-крестовика?
Если мы про это спросим,
Нам ответит цифра… (8)

– Сколько ты имеешь ножек? –
Осьминога тихо спросим.
Тот ответить нам не сможет,
Но и так мы знаем – … (8)

Цифра с виду как игрушка —
Неваляшка—погремушка.
Не удариться ей о земь.
Всем понятно — это… (8)

Эта циферка с секретом.
И зимой, и жарким летом
Различишь едва-едва,
Где в ней ноги, голова.
(8)

Ах, какая запятая
На листе лежит большая!
Можно ею даже мерить,
Просто это цифра… (9)

Вот так циферка, взгляни,
У нее и фокус есть –
Ты её переверни,
И получишь цифру шесть!
(9)

Сколько в дюжине пиратов,
Если три ушли куда-то,
Месяцев в году без лета,
Исполнителей нонета,
Жизней у бродячей кошки,
И в десятке мух без мошки?
Не ищи ответ нигде, ведь
Есть ответ у цифры… (9)

Цифра шесть перевернулась,
Новой цифрой обернулась!
(9)

Цифра эта так хитра,
Ты её переверни,
И начнутся чудеса —
Цифру шесть увидишь ты!
(9)

Нолик, стань за единицей,
За своей родной сестрицей.
Только так, когда вы вместе,
Называть вас будут … (10)

Сколько ночью звезд на небе,
Сколько хлебных крошек в хлебе,
Сколько капелек в дожде,
Сколько рыб живут в воде,
Сколько ног у многонога?
Очень-очень-очень…
(много)
 

Он похож на колобок,
Он пузат и круглобок.
На него похожа Кошка,
Если сложится в клубок.

Сколько лет в яйце цыпленку,
Сколько крыльев у котенка,
Сколько в алфавите цифр,
Сколько гор проглотит тигр,
Сколько мышка весит тонн,
Сколько в стае рыб ворон,
Сколько зайцев съела моль,
Знает только цифра…

Я оранжевый овал
На листе нарисовал.
У него большая роль,
Так как это цифра…

Встали цифры, как отряд,
В дружный числовой свой ряд.
Первой по порядку роль
Нам сыграет цифра…

Могу назвать его мячом,
А хочешь дыркой назовем,
А можно бубликом,
Почти что кругленьким.
Но как его не назовем,
Он называется..

Ты уже нарисовал
Аккуратненький овал?
Нету проще ничего:
Он похож на букву “О”.

Он похож на букву О,
Он не значит ничего.
Но любую цифру враз
Увеличит в десять раз.

Мячик скачет по страницам.
Ищет он свою сестрицу,
Что имеет вид кольца –
Без начала и конца.

Сколько солнышек за тучкой,
Сколько стержней в авторучке,
Сколько у слона носов,
Сколько на руке часов?
Сколько ног у мухомора
И попыток у сапера,
Знает и гордится собой,
Цифра-столбик… (единица).

На стебелёк она похожа,
Стоит почтенно, как вельможа.
Прямая, ровная всегда,
После нуля идёт она.

Стоит она среди листа
Одна, когда тетрадь пуста.
Задрав свой нос до потолка,
Она бранит ученика.
И словно цапля средь болот
Его за лень его клюет.
Хоть у нее одна нога
Она стройна, горда, строга.

Ни журавль то, ни синица.
А всего лишь…(единица)

С хитрым носиком сестрица
Счет откроет …(единица)

Десять лун бы появилось –
И тогда бы осветилась
Ночь как день! Но, жаль, луна
Светит нам всегда …(одна)

Сколько ушек на макушке,
Сколько ног у пол-лягушки,
Сколько у сома усов
У полюсов планеты,
Сколько в целом половинок,
В паре – новеньких ботинок,
И передних лап у льва
Знает только цифра… (два)
Светит солнце, пруд цветет,
Лебедь по нему плывет,
Ближе он подплыл едва —
Оказалась цифра …

Людям всем дано от Бога
По одной лишь голове!
Ну, а что руки, а ноги?
Их у каждого по …(две)

Что скользит по светлой глади
Ученической тетради
Белым лебедем прекрасным,
Ставшим от позора красным
За бездельника, плутишку
Непослушного мальчишку?
То, за что его ругают
И конфет в обед лишают.

С легким росчерком пера
Появилась цифра…

Лебедь плавает в тетрадке,
Значит что-то не в порядке.
Если ты совсем Незнайка,
Эту цифру получай-ка.

Сколько месяцев в зиме,
В лете, в осени, в весне,
Сколько глаз у светофора,
Баз на поле для бейсбола,
Граней у спортивной шпаги
И полос на нашем флаге,
Что нам кто ни говори,
Знает правду цифра…

У меня есть две конфетки,
Дам одну сестренке Светке.
Я не жадный, и за это
Дал мне папа две конфеты!
И теперь конфеток стало,
Даже больше, чем сначала!

Вот так чудо! Ну-ка, ну-ка,
Ты получше посмотри –
Это вроде бы и буква,
Но еще и цифра …

Эту цифру угадай-ка!
Она большая зазнавай-ка.
Единицу сложишь с двойкой,
И получишь цифру …(три)

Эта цифра просто чудо.
У нее родня повсюду.
Даже в алфавите есть
У нее сестра-близнец.

Сколько лапок у мангуста,
Лепестков в цветке капусты,
Пальцев на куриной ножке
И на задней лапе кошки,
Рук у Тани вместе с Петей
И сторон всего на свете
Да и океанов в мире,
Знает циферка… (четыре)

К трем лягушкам у болота
Прибежали два енота,
Прискакала тетя жаба
И пришла наседка Ряба.
Сколько в камышах болотных
Оказалось земноводных?

Я у бабушки была, –
У нее во всей квартире
Три огромные стола,
Ног у каждого – …

Кто-то ночью старый стул
Спинкой вниз перевернул.
И теперь у нас в квартире
Стал он цифрою …

Я сестру свою баюкал,
Утешал я Ксюшу,
Положил я к ней трех кукол,
Зайчика из плюша.
Сколько же всего игрушек
У моей сестренки Ксюши?

То ли цифра, то ли вилка,
То ли развилка двух дорог.
В ученической тетради
Знаю точно – все ей рады.

Сколько пальцев на руке
И копеек в пятачке,
У морской звезды лучей,
Клювов у пяти грачей,
Лопастей у листьев клена
И углов у бастиона,
Про все это рассказать
Нам поможет цифра…

Циферка на крюк похожа,
Что на автокране.
И на двоечку похожа,
Если вверх ногами.

На руке малышка Лена
Любит считать пальчики!
У нее, на удивленье,
Каждый раз выходит …

Смотрит мама с нетерпеньем
На страницы дневника.
Ждет заветную оценку
У сынка-озорника.
Но опять одни четверки.
Нет красавицы…(пятерки)

Если два перевернуть
И внимательно взглянуть,
Так и сяк взглянуть опять,
То получим цифру …

За окном сидели птички,
Голубь, дрозд и три синички.
Спросим мы учеников
И прилежных учениц:
«Кто ответить нам готов,
Сколько за окошком птиц?»

Сколько букв есть у дракона
И нулей у миллиона,
Разных шахматных фигур,
Крыльев у трех белых кур,
Ног у майского жука
И сторон у сундука.
Коль не можем сами счесть,
Нам подскажет цифра…

Циферка, как бегемотик,
Круглый у неё животик,
Шейку ловко изгибает,
За пятёркою ступает.

Отгадайте-ка, ребятки,
Что за цифра-акробатка?
Если на голову встанет,
Ровно на три больше станет.

А моей подруге Тоне
Довелось на пони сесть,
И у Тони вместе с пони
Стало ножек сразу …(шесть)

Глядела цифра в зеркало
И о сестре мечтала.
Но только одного свойства
Его видать не знала.
И получила двойника.
Как капелька водицы
Сестра похожа на нее.
Да только вниз косица. (шесть и девять)

Если навесной замок
Вверх поднимет хоботок,
То тогда увидим здесь
Не замок, а цифру …

У Надюши пять тетрадок,
Кляксы в них и беспорядок.
Нужен Наде черновик.
Вася, первый ученик,
Дал еще тетрадку Наде
Сколько у нее тетрадей?

Сколько в радуге цветов,
Дней в неделе у китов.
Гномиков у Белоснежки,
Братьев-близнецов у пешки,
Нот, что знают даже дети,
И всего чудес на свете,
Разобраться с этим всем
Нам поможет цифра…

Сестры – белочки сидели
Вшестером в дупле на ели.
К ним еще одна примчалась
От дождя она спасалась.
Все теперь сидят в тепле.
Сколько белочек в дупле?

На косу она похожа,
Но косить траву не может —
Не наточена совсем
И не косит цифра …

Едут с ярмарки купцы –
Удалые молодцы:
Фрол, Степан, Панкрат, Тимошка,
Ванька, Сенька и Антошка.
Мы пожмем им руки всем,
Превратив их в цифру…

Есть пословица у нас
И она известна всем –
Отрезаешь только раз,
А отмерить надо …

Я – цифра, меньше десяти.
Меня легко найти.
Но если букве «я» прикажешь встать рядом,
Я все: отец, и ты, и дедушка, и мать…

Вьется по ветру коса,
А по спинке полоса.

Сколько на море ветров,
И копыт у двух ослов,
Щупалец у осьминога,
И клыков у пары догов?
Сколько ног у паука,
Паука-крестовика?
Если мы про это спросим
Нам ответит цифра…

Два кружочка рядом встали —
Что за дивные очки?
Поверни хоть вверх ногами,
Одинаковы они!

Два кольца, но без конца,
В середине нет гвоздя.
Если я перевернусь,
То совсем не изменюсь.
Ну, какая цифра я?

– Сколько ты имеешь ножек? –
Осьминога тихо спросим.
Тот ответить нам не сможет,
Но и так мы знаем – …

Друг на друге два кольца,
Удалых два молодца.
В числовой их ряд попросим
И получим цифру…

Пять мальчишек было в классе,
В класс вошел еще и Вася,
А потом Илья и Миша.
Сколько же теперь мальчишек?

Цифра с виду как игрушка –
Неваляшка-погремушка.
Не удариться ей о земь.
Всем понятно – это…

Сколько в дюжине пиратов,
Если три ушли куда-то,
Месяцев в году без лета,
Исполнителей нонета,
Жизней у бродячей кошки,
И в десятке мух без мошки?
Не ищи ответ нигде, ведь
Есть ответ у цифры…

Глядела цифра в зеркало
И о сестре мечтала.
Но только свойства одного
Его видать не знала.
И получила двойника.
Как капелька водицы
Сестра похожа на нее.
Да только вниз косица.

Вот так циферка, взгляни,
У нее и тайна есть –
Ты ее переверни,
И получишь цифру шесть!

Отгадайте-ка, ребятки,
Что за цифра-акробатка?
Если на голову встанет,
Ровно на три меньше станет.

Эта цифра чуть больше восьмерки
Но пока ещё не десятка
Это сумма шестерки и тройки
Кто способен решить загадку?

Нолик, стань за единицей,
За своей родной сестрицей.
Только так, когда вы вместе,
Называть вас будут …

Вот единица, а рядом ноль,
Стоит важный как король.
Если к единице ноль прислонится,
В какое число он превратится?

Палочка-тростинка,
А сзади колесо.
Попробуй угадай-ка,
Как назвать число?

Учит брат меня считать
-Сколько будет 5 и 5
Я подумала немножко
И сказала:
– Две ладошки!
А сколько будет 5 и 5
Надо пальцы посчитать.

Повело опять Егорке,
У реки сидел не зря
6 карасиков в ведерке
И 4 пескаря.
Сколько же всего, друзья?

3 синички летят,
2 синички сидят,
5 синичек пели песни,
И всегда бывали вместе.
Сколько птичек сосчитай
И ответ скорее дай.

Загадки с цифрами | Kidside.ru

В предыдущем посте мы разбирали загадки о цифрах. В этой подборке собраны загадки, в тексте которых упоминаются цифры.

-1-
  1.  Стоит Антошка на одной ножке; его ищут, а он не откликается (гриб).
  2.  На ноге стоит одной, крутит-вертит головой. Нам показывает страны, реки, горы, океаны (глобус).
  3. На длинной ножке, застыв до поры, отдыхает палочка после игры (единица).
  4.  У кого одна нога, да и та без башмака? (гриб).
  5. Много рук, нога — одна (дерево).
  6. Стоят в один ряд острые пальчики — цап — царапки: подбирай охапки! (грабли).
  7.  На одной ноге кружится, беззаботна, весела. В пестрой юбке танцовщица, музыкальная … (юла).
  8. На одной ноге стоит, в воду пристально глядит. Тычет клювом наугад, ищет в речке лягушат. На носу повисла капля. Узнаете? Это … (цапля).
  9. В лесу на одной ножке выросла лепешка (гриб).

-2-
  1.  Два братца пошли в реку купаться (ведра).
  2. Два кольца, два конца, посередине — гвоздик (ножницы).
  3. Два домика — теплушки подарены Танюшке (варежки).
  4. Шея такая длинная, хвост крючком… И не секрет, любит всех она лентяев, а ее лентяи — нет! (двойка).
  5. Есть совсем другая птица: если сядет на страницу, то с поникшей головой, я возвращаюсь домой (двойка).
  6. Сговорились две ноги делать дуги и круги (циркуль).
  7. На ночь два оконца сами закрываются, а с восходом солнца сами открываются (глаза).
  8. Есть у каждого лица два красивых озерца. Между ними есть гора. Назови их, детвора. (глаза).
  9. Между двух светил, посередине — один (нос).
  10. Не мотылек, не птичка, а держит две косички (бант).
  11. Две сестренки, две плетенки из овечьей шерсти тонкой. Как гулять, так надевать, чтоб не мерзли пять да пять! (варежки).
  12. Две стройные сестрицы в руках у мастерицы. Весь день ныряли в петельки … И вот он шарф для Петеньки (спицы).
  13. Две сестрицы друг за другом пробегают круг за кругом: коротышка только раз, та, что выше — каждый час! (стрелки часов).
  14. Бежит-гудит, в два глаза глядит, а станет — ярко-красный глазок глянет! (автомобиль).
  15. Это конь не ест овса. Вместо ног — два колеса. Сядь верхом и мчись на нем, только лучше правь рулем (велосипед).
  16. У него два колеса и седло на раме, две педали есть внизу, крутят их ногами (велосипед).
  17. Два коня у меня, два коня. По воде они возят меня, а вода тверда, словно каменная! (коньки).
  18. Две полоски на снегу оставляю на бегу. Я лечу от них стрелой, а они опять за мной (лыжи).
  19. Две новые кленовые подошвы двухметровые: на них поставил две ноги — и по большим снегам беги (лыжи).
  20. Два братца через мать, друг на друга глядят (берега).
  21. Два соболя лежат хвостами друг к другу (брови).
  22. Две глядят, да двое слушают (глаза и уши).
  23. Два родных брата: одного всякий видит, да не слышит; другого все слышат, да не видят (молния и гроза).
  24.  Вот гора, а у горы две глубокие норы. В этих норах воздух бродит, то заходит, то выходит (нос).
  25. Два близнеца, два братца на нас верхом садятся (очки и нос).
  26. Двое в небе ходят кругом, но не видятся друг с другом (солнце и луна).
  27. Едет он на двух колесах, не буксует на откосах. И бензина в баке нет. Это мой … (велосипед).
  28. Он на вокзале всегда есть, к нему подходят поезда. Двойное Р содержит он и называется … (перрон).
  29.  На большие расстояния он мчится без опоздания. Пишется в конце два С, называется … (экспресс).
  30. Загадка эта нелегка: пишусь всегда через два К. И мяч, и шайбу клюшкой бей, а я называюсь … (хоккей).
  31. В конце два Л пиши. А как зовут меня реши: без мастера граненным стал блестящий правильный … (кристалл).
  32. Две сестры: одна светлая, другая темная (день и ночь).

-3-
  1.  Есть спина, а не лежит никогда. Есть четыре ноги, а не ходят и три. Сам всегда стоит, а всем сидеть велит (стул).
  2. Я стою на трех ногах, ноги в черных сапогах. Зубы белые, педаль. Как зовут меня? (рояль).
  3. Входишь в одну дверь, а выходишь из трех. Думаешь, что вышел, а на самом деле — вошел (рубашка).
  4. Треугольная доска, а на ней три волоска. Волосок — тонкий, голосок — звонкий (балалайка).
  5. Три братца пошли на реку купаться. Два купаются, третий на берегу валяется. Искупались — вышли, на третьем повисли (ведра и коромысло).
  6. Возле леса на опушке их трое живет в избушке. Там три стула и три кружки, три кровати, три подушки. Угадайте без подсказки, кто герои этой сказки? (Машенька и три медведя).
  7. Трое одним лугом пашут (пальцы пишут).
  8. У него цветные глаза, не глаза, а три огня, он по очереди ими сверху смотрит на меня (светофор).
  9.  Вот стоит на улице, в длинном сапоге, трехглазое чудище на одной ноге. Запылал у чудища изумрудный глаз — значит, можно перейти улицу сейчас (светофор).

-4-
  1.  Четыре братца под одной крышей стоят (стол).
  2. Хоть у нас 4 ножки, мы не мышки и не кошки. Хоть мы все имеем спинки, мы не овцы и не свинки. Мы не кони, хоть на нас вы садились сотни раз (стулья).
  3.  Под крышей 4 ножки, а на крыше суп да ложки (стол).
  4. На 4 ногах стою, ходить я вовсе не могу. Когда ты устанешь гулять, ты можешь сесть и отдыхать (стул).
  5. На 4 ноги надевали сапоги. Перед тем как надевать, стали обувь надувать (шины).
  6. Шевелились у цветка все четыре лепестка. Я сорвать его хотел, он вспорхнул и улетел (бабочка).
  7. Ежегодно приходят к нам в гости: один седой, другой молодой, третий скачет, а четвертый плачет (времена года).
  8. Четыре крыла, а не птица; крыльями машет, а не с места (мельница).
  9. Вспушит она свои бока, свои четыре уголка. И тебя, как настанет ночь, все равно к себе притянет (подушка).
  10. Четыре грязных копытца залезли прямо в корытце (поросенок).
  11. Живет между камнями голова с четырьмя ногами (черепаха).
  12. Кто году в четыре раза переодевается? (Земля)
  13.  В году у дедушки четыре имени (зима, весна, лето, осень).
  14. Говорит она беззвучно, но понятно и не скучно. Ты беседуй чаще с ней — станешь вчетверо умней (книга).
  15. Четыре ноги, да не зверь. Есть перья, да не птица (кровать с подушкой).
  16. Два брюшка, четыре рожка (подушка).
  17. Четыре уха, а перьев не сосчитать (подушка).

-5-
  1.  Есть, друзья, такая птица: если сядет на страницу, очень рад бываю я, а со мною вся семья (пятерка).
  2. Пятерка братьев неразлучна, им вместе никогда не скучно. Они работают пером, пилою, ложкой, топором (пальцы).
  3. У пяти братьев одна работа (пальцы).
  4. У двух матерей по пяти сыновей, одно имя всем (пальцы).
  5. Как только она отправляется зимой гулять, жильцы дома в вселяются, и в каждый — ровно пять! (перчатки).
  6. 5 пальцев, как у людей, но пальцы у нее без ногтей (перчатки).
  7.  5 мешочков шерстяных — греются братишки в них (перчатки).
  8. На пяти проводах отдыхает стая птах (ноты)
  9. Чтоб не мерзнуть, пять ребят в печке вязаной сидят (пальцы в варежке).
  10. Пять ступенек — лесенка, на ступеньках — песенка (ноты).

-6-
  1.  Если на голову встанет, ровно на три больше станет (шесть).
  2.  Черен, да не ворон. Рогат, да не бык. Шесть ног без копыт. Летит жужжит, упадет — землю роет (жук).
  3.  На дворе переполох, с неба сыплется горох. Съела 6 горошин Нина, у нее теперь ангина (град).
  4. 6 для ног, 2 головы, один хвост. Кто это? (всадник на коне).

-7-
  1.  Ежедневно в 7 утра, я трещу: пора вставать! (будильник).
  2. Есть 7 братьев: годами равные, именами разные (дни недели).
  3. Братьев этих ровно 7. Вам они известны всем. Каждую неделю кругом ходят братья друг за другом. Попрощается последний — появляется передний (дни недели).
  4.  Всю жизнь я ношу два горба, имею два желудка! Но каждый горб — не горб, амбар! Еды в них на семь суток! (верблюд)
  5. Пять щенят, да мама-лайка. Ты попробуй, сосчитай-ка! (6)
  6. Приказало солнце: » Стой, семицветный мост дугой!» (радуга)
  7. Нас отара, 7 баранов, защищаем от буранов (шуба).
  8. Один семерых полонил (паук).

-8-
  1.  Чудесный Дом — бегунок на своей восьмерке ног. День-деньской в дороге: бегает аллейкой по двум стальным змейкам (трамвай).
  2. Я так мила, я так кругла, я состою из двух кружочков. Как я рада, что нашла себе таких, как вы дружочков (8).
  3.  Ты со мною не знаком? Я живу на дне морском. Голова и 8 ног, вот и весь я — ….(осьминог).
  4.  8 ног, как 8 рук, вышивают шелком круг. Мастер в шелке знает толк. Покупайте, мухи, шелк! (паук).

-9-
  1.  Отгадайте-ка, ребятки, что за цифра акробатка? Если на голову встанет, ровно на три меньше станет (9).

-10-
  1. Твои помощники — взгляни — десяток дружных братцев. Как славно жить, когда они работы не боятся (пальцы).
  2. Ежик вырос в десять раз, получился … (дикобраз).
  3.  Проживают в умной книжке хитроумные братишки. 10 их, но эти братья сосчитают все на свете (цифры).
  4. Есть у меня работники, во всем помочь охотники. Живут не за стеной — день и ночь со мной: целый десяток, верных ребяток! (пальцы).
  5. На десятки верст — разноцветный мост. Только вот по нему не шагать никому (радуга).

Числа больше десяти
  1. 70 одежек, а все без застежек (капуста).
  2. На странице букваря 33 богатыря. Мудрецов богатырей знает каждый грамотей (буквы).
  3. Сели на страничке 33 сестрички. Сели рядом — не молчат, нам загадки говорят (буквы).
  4. Кулик — не велик, целой сотне велит: то сядь, да учись; то встань, разойдись (школьный звонок).
  5. У меня знакомых — тьма. Не могу счесть их сама, потому что кто пройдет, тот и руку пожмет мне (дверь).
  6. Сотнями глаз во все стороны глядит (наперсток).
  7.  В два ряда дома стоят — 10, 20, 100 подряд. И квадратными глазами друг на друга глядят (улица).
  8. 12 братьев равно и называются разными делами занимаются (месяцы года).
  9.  Рассыпался горох на семьдесят дорог: никто его не подберет (град).
  10. Золотист он и усат. В ста карманах — 100 ребят (колос).
  11. Лето целое старалась — одевалась, одевалась… А как осень подошла, нам одежки отдала. Сотню одежонок мы сложили в бочонок (капуста).
  12. Тысяча братьев одним поясом подпоясаны (колосья в снопу).
  13. Один пастух 1000 овец пасет (месяц и звезды).
  14. Золотое решето черных домиков полно. Сколько черненьких домков, столько беленьких жильцов (подсолнух).
  15. Сто березовых солдат, взявшись за руки, стоят. Днем и ночью, круглый год: охраняют огород (забор).

Домашнее задание по математике требует творческого подхода. Выберите ту загадку, к которой легко нарисовать иллюстрацию, тогда ваша книжка про цифры удостоится отдельной похвалы!

Понравилось это:

Нравится Загрузка…

Загадки в числах. Загадки в цифрах. Один, два, три, четыре, пять.

ЗАГАДКИ ДЛЯ ДЕТЕЙ

Вопросы на логику с подвохом

Загадки в числах.

Загадки в числах. Вариант №1.

Десять дружных братиков,
Славных математиков,
Сосчитают всё на свете.
Вы знакомы с ними, дети?

(Цифры, числа)

Могу назвать его мячом,
А хочешь, дыркой нозовём,
А можно бубликом,
Почти что кругленьким.
Но как его не назовём,
Он называется …

(Ноль)

Я умею делать стойку,
По канату я хожу,
За собою цифру двойку
На верёвочке вожу.

(Один)

Вот – шея гибкая,
Вот – тело,
К воде склонилась голова.
Ты птицу нарисуй умело
И это будет цифра …

(Два)

Это заклинатель змей,
Вышел с дудочкой своей.
Перед ним танцует змейка –
Хвост крючком, дугою шейка.
Ты на змейку посмотри –
Да ведь это цифра …

(Три)

Делал Егорка
С мамой уборку,
Опрокинул стул
В квартире,
Стал похож он на …

(Четыре)

Отгадайте-ка, друзья,
Что за цифрой буду я?
Ручку вправо повернула,
Ножку круто изогнула?

(Пять)

Рогалик к бублику приклей,
Да только есть его не смей.
Не крендель это, чтобы есть.
А просто – это цифра…

(Шесть)

Размышлял я полчаса –
Цифра это иль коса.

(Семь)

Ну и цифра – шар на шаре!
Мы зимой её встречали.

(Восемь)

Отгадайте-ка ребятки,
Что за цифра-акробатка?
Если на голову встанет,
Ровно на три меньше станет.

(Девять)

С нулём гуляет единица.
Не может с ним наговориться.

(Десять)

Загадки в числах. Вариант №2.

Двенадцать братьев
Друг за другом ходят,
Друг друга не обходят.

(Месяцы)

Выпускает он листы
Широченной широты.
Держатся на стеблях крепких
Сто плодов шершавых цепких:
Если их не обойдёшь,
На себе их все найдёшь.

(Репейник)

Сто гостей, сто постель. У каждого гостя своя.

(Брёвна в стенах избы, проложенные мхом)

Четыре сестрицы под одной фатицей.

(Стол)

Пять пальцев, как у людей, но пальцы у нас без ногтей.

(Перчатка)

Один выход, три входа.

(Рубашка)

Два братца в воду глядятся,
Вовек не сойдутся.

(Берега)

Ползёт паук –
Восемь ног, пара рук…
В руках клещи,
В глазах испуг.

(Краб)

У рыжей плутовки
Тридцать три уловки.
Без них – трудно прожить,
Трудно корм добыть.

(Лиса)

Загадки в числах. Вариант №3.

Стоит дуб,
На нём двенадцать гнёзд,
В каждом гнезде
По четыре яйца,
В каждом яйце
По семь цыплят.

(Год, месяцы, недели, дни)

Нам в дождь и зной
Поможет друг,
Зелёный и хороший –
Протянет нам он сотни рук
И тысячи ладошек.

(Дерево)

Под крышей – четыре ножки. А на крыше – суп да ложки. Что это такое?

(Стол)

Пять чуланов, одна дверь.

(Перчатка)

Два бочонка –
Днём полны, а ночью пусты.

(Сапоги.)

Пять морских ежей схватила
Я в подводной глубине
И, спокойно проглотив их,
Ярче вспыхнула на дне.

(Морская звезда)

Если есть у кисочки,
На ушах две кисточки
Эту киску берегись,
Потому что это …

(Рысь)

Весит более трёх тонн,
“Речным конём” зовётся он.

(Гиппопотам)

Тот мальчишка не велик,
Но сдаваться не привык…
Поднатужился дружок:
Раз-два-три! Ещё разок!..
Ветка сдвинулась – ура!
Вот теперь домой пора.
Не робея тащит груз
По опушке леса
Раз в пятнадцать-двадцать больше
Собственного веса.

(Муравей)

Что же это за медведь:
Не умеет он реветь,
Ест бамбук он на обед,
И в горах живёт сто лет?

(Панда)

Загадки в числах. Вариант №4.

Каждый день и каждый год,
Путник движется вперёд.
Не свернёт куда-нибудь,
Не присядет отдохнуть.
И проходит всякий раз
Шестьдесят минут за час.

(Час)

Стоит Антошка
На одной ножке
Его ищут, а он молчит.

(Гриб)

Есть спина, а не лежит; четыре ноги, а не ходит, но всегда стоит и всем сидеть велит.

(Стул)

Пять пальцев, ни костей, ни мяса, ни ногтей.

(Перчатка)

Семьсот ворот до один вход.

(Сети рыболовные)

Носит серенький жилет,
Но у крыльев чёрный цвет.
Видишь, кружат двадцать пар,
И кричат:
– Карр! Карр! Карр!

(Ворона)

Восемь ног, как восемь рук,
Вышивают шёлком круг.
Мастер в шёлке знает толк,
Покупайте, мухи, шелк!

(Паук)

Твои помощники – взгляни –
Десяток дружных братцев.
Как славно жить, когда они
Работы не боятся.

(Пальцы)

У тётушки Фелицы
Есть четыре сестрицы.
Из них две-то хвалятся:
“Мы делать горазы!”
А другие хвалятся:
“Мы ходить горазды!”

(Руки, ноги)

Есть у каждого лица
Два красивых озерца.
Между ними есть гора.
Назови их, детвора.

(Глаза)

Числа в загадках. Вариант №5.

Я вам скажу заранее,
Она известна всем.
Не делится в названии,
Но делится на семь.

(Неделя)

Дом открыт со всех сторон.
В доме – тысяча колонн.
Над колоннами – шатры,
Под колоннами – ковры.
Тут живут и на коврах,
И в колоннах, и в шатрах.

(Лес)

На четырёх ногах, а не зверь; у неё пух и перья, а не птица; есть душа, но не всегда.

(Кровать, постель)

Пять мальчиков, пять амбарчиков. У каждого мальчика свой амбарчик.

(Пальцы, перчатка)

Два арапа, родные брата, ростом по колено, везде с нами гуляют и нас защищают.

(Сапоги)

Этот зверь, себе на грех,
Дорогой имеет мех.
Двести сорок три стрелка
День и ночь палят в зверька.

(Песец)

Кто дважды родится:
В первый раз – гладенький,
Во второй раз – пухленький?

(Птицы)

Сотни врагов в лесу окружают
Волк и медведь на него нападают,
А для защиты два страшных клыка
И нрав свирепый, как у быка.

(Кабан)

Вот гора, а у горы
Две глубокие норы.
В этих норах воздух бродит,
То заходит, ты выходит.

(Нос)

У неё семь дырок есть:
Эта – чтобы пить и есть,
Этими двумя ты дышишь,
Этими двумя ты слышишь,
А ещё двумя читаешь,
Звёзды на небе считаешь.

(Голова)

Загадки в числах. Вариант №6.

Два кольца, два конца,
А по середине гвоздик.

(Ножницы)

Сто одёжек
И все без застёжек.

(Капуста)

Кто в году
Четыре раза переодевается.

(Земля)

На первую ступеньку
Встал парень молодой,
К двенадцатой ступеньке
Пришёл старик седой.

(Год)

Что за птицы пролетают?
По семёрке в каждой стае.
Вереницею летят,
Не воротятся назад.

(Неделя, дни недели)

Вышел старик-годовик,
Махнул рукавом.
И полетели двенадцать птиц:
У каждой птицы
По четыре крыла,
В каждом крыле
По семь перьев,
Каждое перо
С одной стороны чёрное,
А с другой – белое.

(Год, месяцы, недели, дни)

Четвёрку такую любой назовёт,
Погоду меняет она круглый год:
Мороз – на прохладу, прохладу – на зной,
Зной вновь на прохладу и дождь проливной.

(Времена года)

Вышли два Пахома
Из одного дома.
И от порога
Им врозь дорога:
Одному – вниз,
Другому – вверх.

(Корень и стебель)

Под одной крышей четыре брата живут.

(Стол)

По дороге я шёл, две дороги нашёл, по обеим пошёл.

(Штаны)

ЗАГАДКИ ДЛЯ ДЕТЕЙ

Загадки про Цифры – Коллекция Загадок

Загадка № 9180

Дата: 31.10.2015, 14:50

Горело 5 электрических лампочек,
3 лампочки выключили.
Сколько лампочек осталось?

Ответ: пять


Загадка № 9179

Дата: 31.10.2015, 14:49

Ты да я, да мы с тобой.
Много ли всех?

Ответ: двое


Загадка № 9178

Дата: 31.10.2015, 14:48

Если эту цифру в школе получить,
То родители вас дома наругают.
С лебедем её легко сравнить,
Про цифру эту все детишки знают?

Ответ: два


Загадка № 9177

Дата: 31.10.2015, 14:48

Кто-то ночью старый стул
Спинкой вниз перевернул.
И теперь у нас в квартире
Стал он цифрою…

Ответ: четыре


Загадка № 9176

Дата: 31.10.2015, 14:47

Эта цифра так похожа
На букву алфавита О.
Но без цифорок других
Она не значит ничего.

Ответ: ноль


Загадка № 9175

Дата: 31.10.2015, 14:47

Сколько раз в дверь нужно постучать?
Сколько будет двойка плюс один?
Поскорее нужно это отгадать,
Эта цифра с буквой З — один в один.

Ответ: три


Загадка № 9174

Дата: 31.10.2015, 14:44

Встали цифры, как отряд,
В дружный числовой свой ряд.
Первой по порядку роль
Нам сыграет цифра…

Ответ: ноль


Загадка № 9173

Дата: 31.10.2015, 14:44

На стебелёк она похожа,
Стоит почтенно, как вельможа.
Прямая, ровная всегда,
После нуля идёт она.

Ответ: один


Загадка № 9172

Дата: 31.10.2015, 14:43

На букву О она похожа, кругла собой.
Может в паре постоять с циферкой любой.
Не имеет ни начала, ни конца,
Носит гордое название нуля.

Ответ: ноль


Загадка № 9171

Дата: 31.10.2015, 14:42

Не забьешь так просто гол,
На воротах стоит кол.
И в бою с ним не сразиться,
Это цифра…

Ответ: единица


Загадка № 9170

Дата: 31.10.2015, 14:42

Гибко шейку изгибает,
И красива, и стройна,
Ловко хвостик поднимает,
Что за цифра? Цифра…

Ответ: два


Загадка № 9169

Дата: 31.10.2015, 14:41

Как поётся в песенке одной:
Дважды два — такая будет цифра.
Перед пятёркой юбилейной, золотой,
Стоит она… Ну, кто ответит быстро?

Ответ: четыре


Загадка № 9168

Дата: 31.10.2015, 14:41

Светит солнце, пруд цветет,
Лебедь по нему плывет,
Ближе он подплыл едва —
Оказалась цифра…

Ответ: два


Загадка № 9167

Дата: 31.10.2015, 14:40

Необычна цифра эта,
Круглолица, так и знай.
И на букву З похожа,
Что за цифра? Отгадай!

Ответ: три


Загадка № 9166

Дата: 31.10.2015, 14:40

У отличника повсюду
Эту цифру в дневнике найти.
Ещё одну подсказку не забуду —
Перед ней стоит шестёрка на пути.

Ответ: пять


Загадка № 9165

Дата: 31.10.2015, 14:40

Букву З я обведу,
К цифрам в гости приведу.
Ты внимательней смотри —
Получилась цифра…

Ответ: три


Загадка № 9164

Дата: 31.10.2015, 14:39

Циферка на крюк похожа,
Что на автокране.
И на двоечку похожа,
Если вверх ногами.

Ответ: пять


Загадка № 9163

Дата: 31.10.2015, 14:39

А с этой цифрой лишь сплошные суеверия,
И так обидно: ну, за что ей это всё?
И место в счёте нам внушает лишь доверие:
Между пятёркой и семёркою оно.

Ответ: шесть


Загадка № 9162

Дата: 31.10.2015, 14:39

Решили фильм про цифры снять,
Стали камеры включать.
Откуда буква Ч в эфире?
Не бойтесь! Цифра я!..

Ответ: четыре


Загадки про цифры | Картотека (младшая, средняя, старшая, подготовительная группа) на тему:

Загадки про цифры

Без них прожить друзья нельзя
Надо посчитать машинки,
Карандаши, заколки,
Не напишем день рождения,
Не узнаем номер автобуса,
Сдачу нам не дадут,
Не узнаем сколько минут.
Каждый новый день приходит,
Он другое число приводит!

(Цифры)

Проживают в умной книжке
Хитроумные братишки.
Десять их, но братья эти
Сосчитают все на свете.

(Цифры)

Проживают в трудной книжке

Хитроумные братишки. 

Десять их, но братья эти 

Сосчитают все на свете.  

(Числа)

Цифра счет нам открывает,
Тонкий нос её завершает.

(Единица)

С хитрым носиком сестрица
Счёт откроет …

(Единица)

Сотня лун бы появилась – 

И тогда бы осветилась 

Ночь как день! 

Но, жаль, луна 

Светит нам всегда …  

(Одна)

 

Стоит она среди листа

Одна, когда тетрадь пуста.

Задрав свой нос до потолка,

Она бранит ученика.

И словно цапля средь болот

Его за лень его клюет.

Хоть у нее одна нога

Она стройна, горда, строга.

Ни журавль то, ни синица.

А всего лишь…

(Единица)

Высока, как палочка,
И с хвостиком большим,
Всегда повсюду первая — 
Циферка ….
(Один) 

Два орешка бурундук
Положить хотел в сундук.
Только он не утерпел
И один орешек съел.
Сколько у бурундука осталось орешков?
(Один)

Три тигрёнка не грустили,
Шар воздушный смастерили.
Два взлетели на рассвете
И пропали вдруг из глаз.
Коль отнять тигрят нам этих —
Что останется у нас?
(Один)

Пели песню две синицы,
Две подружки, две певицы.
Тут одна из них пропала,
Видно петь она устала.
Здесь подсказка не нужна:
Было две, теперь …
(Одна)

Лебедь плавает в тетрадке,
Значит что-то не в порядке.
Если ты совсем Незнайка,
Цифру эту получай-ка.

(Двойка)

Словно лебедь, чудо-птица,
По морю мчится, как царица,
Как называется она?
Ну, конечно, цифра …
(Два)

Вырос гриб в тени осин,
Он сначала был один.
Тут второй грибок пробился,
Рядом с первым очутился.
Стала их считать сова:
Получилось ровно …
(Два)

Кошка в кубики играла,
Кошка кубик потеряла.
Закружилась голова…
Было три, осталось…
(Два)

А теперь пора бананы
Сосчитать у обезьяны.
Один, два, три, четыре, пять…
Можно три на завтрак дать.
Если три на завтрак дать,
Их уже не будет пять.
Обезьянка, ты права:
Их останется лишь …
(Два)

Что скользит по светлой глади

Ученической тетради

Белым лебедем прекрасным,

Ставшим от позора красным

За бездельника, плутишку

Непослушного мальчишку?

То, за что его ругают

И конфет в обед лишают. 

 С легким росчерком пера

Появилась цифра…

(Два)

 

Людям всем дано от Бога

По одной лишь голове! 

Ну, а руки что, а ноги? 

Их у каждого по …  

(Две)

Цифру эту угадай-ка!
Она большая зазнавай-ка.
Единицу сложишь с двойкой,
И получишь цифру …
(Тройку)

У богатого волчонка
Было кваса два бочонка.
Тут еще один бочонок
Прикатил домой волчонок.
Вместе складывать их стал — 
Сколько он их насчитал?
(Три)

Четыре гусёнка гуляли в саду,
Один искупаться задумал в пруду,
Решил с головой он под воду нырнуть…
А сколько гусят продолжают свой путь?
(Три)

Семь бумажных кораблей
Сделал мастер-воробей.
Подарил четыре мишкам,
Что оставил воробьишкам?
(Три)

Эта цифра просто чудо.

У нее родня повсюду.

Даже в алфавите есть 

У нее сестра-близнец.

(Тройка)

 

Вот так чудо! Ну-ка, ну-ка, 

Ты получше посмотри – 

Это вроде бы и буква, 

Но еще и цифра …  

 (Три)

Кто-то ночью старый стул
Спинкой вниз перевернул.
И теперь у нас в квартире
Стал он цифрою …
(Четыре)

Два мышонка грызли корку,
Два — сырок тащили в норку.
Сколько их у нас в квартире?
Два плюс два — всего …
(Четыре)

Белка шишки собирала,
Собирала и считала.
Забиралась на сосну, 
Прибавляла к трём — одну.
Было шишек три сначала,
А теперь их сколько стало?
(Четыре)

То ли цифра, то ли вилка,

То ли двух дорог развилка.

В ученической тетради

Знаю точно – все ей рады.

(Четверка)

 

Я у бабушки была, – 

У нее во всей квартире 

Три огромные стола, 

Ног у каждого – … 

(Четыре)

Если ДВА перевернуть
И внимательно взглянуть,
Так и сяк взглянуть опять,
То получим цифру …
(Пять)

Три синицы на кормушке.
Две синицы на кадушке.
Мы умеем прибавлять:
Всех синичек будет …
(Пять)

Торговал баран на рынке,
Продавал баран корзинки.
Семь корзинок продавал
И жевал, жевал, жевал …
Двух корзинок больше нет,
Значит, пишем что в ответ?
(Пять)

По дорожке мышка шла,
Девять зёрнышек несла.
Четыре — птичке отдала,
Что в кладовка убрала?
(Пять)

Смотрит мама с нетерпеньем

На страницы дневника.

Ждет заветную оценку 

У сынка-озорника.

Но опять одни четверки.

Нет красавицы…

(Пятерки)

 

На руке малышка 

Лена Любит пальчики считать! 

У нее, на удивленье, 

Каждый раз выходит …  

(Пять)

Если навесной замок
Вверх поднимет хоботок,
То тогда увидим здесь
Не замок, а цифру…

(Шесть)

Васька — ловкий рыбачок — 
Ловит рыбок на крючок.
Трёх поймал он на рассвете,
Трёх поймал в вечерний час.
Три плюс три любой ответит
Сколько рыбок есть у нас?
(Шесть)

Глядела цифра в зеркало

И о сестре мечтала.

Но только свойства одного

Его видать не знала.

И получила двойника.

Как капелька водицы

Сестра похожа на нее.

Да только вниз косица.

(Шесть и девять)

 

А моей подруге Тоне 

Довелось на пони сесть,

И у Тони вместе с пони 

Стало ножек сразу …  

(Шесть)

На косу она похожа,
Но косить траву не может —
Не наточена совсем
И не косит цифра …

(Семь)

Вьется по ветру коса,

А средь спинки полоса.

(Семь)

 

Есть пословица у нас 

И она известна всем – 

Отрезаешь только раз, 

А отмерить надо …  

(Семь)

 

Я – цифра, меньше десяти.  

Меня легко найти.  

Но если букве «я» прикажешь рядом встать,  

Я всё: отец, и ты, и дедушка, и мать… 

(Семь)

Эта циферка с секретом.
И зимой, и жарким летом
Различишь едва-едва,
Где в ней ноги, голова.

(Восьмерка)

Цифра с виду как игрушка –

Неваляшка-погремушка.

Не удариться ей о земь.

Всем понятно – это…

(Восемь)

 

– Сколько ты имеешь ножек?

– Осьминога тихо спросим. 

Тот ответить нам не сможет,

Но и так мы знаем – …  

(Восемь)

Цифра шесть перевернулась,
Новой цифрой обернулась!

(Девять)

Вот так циферка, взгляни, 

У нее и фокус есть – 

Ты её переверни, 

И получишь цифру шесть!  

(Девять)

Нолик, стань за единицей,
За своей родной сестрицей.
Только так, когда вы вместе,
Называть вас будут …

(Десять)

Рано утром у реки
Распустились васильки.
Пять — на левом берегу.
Пять — на правом берегу.
Я цветочки-василёчки
Сосчитать легко смогу.
Потому что пять да пять
Будет сколько, как не знать?
(Десять)

Сколько вместе будут весить 

Две гантели с цифрой пять? 

Да, не надо и гадать – 

Не кило, а целых …  

(Десять)

Нолик, стань за единицей,
За своей родной сестрицей.
Только так, когда вы вместе,
Называть вас будут…

(Десять)

Вот единица, а рядом ноль,
Стоит важный как король.
Если к единице ноль прислонится,
В какое число он превратится?

(Десять)

Он похож на колобок,
Он пузат и круглобок.
На него похожа Кошка,
Если сложится в клубок.

(Ноль)

Скачет мячик по страницам,
Ищет он свою сестрицу.
Что имеет вид кольца — 
Без начала и конца?
(Ноль)

62 Загадки с числами (для взрослых и детей)

Пожалуй, одна из самых сложных загадок – это загадки с числами. Надо действительно остановиться и подумать, идти шаг за шагом, а также понять, что ответ может быть умной игрой слов или необычным поворотом. В этой статье мы выбрали много типов загадок с числами; те, кто полагается на математические навыки, те, которые требуют серьезных вычислений, и те, кто просто использует слова для получения неожиданного результата. У нас есть числовые загадки для детей и взрослых, а также целый раздел с загадками и ответами, перечисленными отдельно, чтобы проверить свои способности к разгадыванию загадок.Запомните своих фаворитов, чтобы поделиться с друзьями и проверить их математические способности.

Загадки с числами для детей

Загадки с числами – хороший способ научить ребенка заниматься математикой, весело проводя время. Младшие дети не поймут, что они учатся, пока они решают нашу коллекцию сложных числовых загадок. Загадки с числами не только забавны для детей, но и имеют много преимуществ. Загадки с числами помогут ребенку:

  • Научитесь получать удовольствие от работы с числами.
  • Расслабьтесь и наслаждайтесь математическими концепциями.
  • Повысьте способность использовать мысленные образы для решения математических задач.
  • Развивайте стратегическое мышление.
  • Обучайтесь и развивайте навыки решения проблем.

Неважно, сколько лет вашим детям, у нас есть ряд загадок, которые они могут разгадать и которые им понравятся.

  • Что становится меньше при переворачивании?

    Номер девять

  • Идет в город человек на своей лошади. Мужчина уехал в город в понедельник.Он пробыл там 3 дня, а затем вернулся в пятницу. Как? Его лошадь не устала, человек не спал.

    Лошадь звали Пятница.

  • Что общего у чисел 11 и 88?

    Они оба выглядят одинаково в перевернутом виде и задом наперёд.

  • Я нечетное число. Забери письмо и я стану ровным. Какой я номер?

    Семь

  • Если двое составляют компанию, а трое – толпа, то какие четыре и пять?

    Девять

  • Маленькая девочка идет в магазин и покупает дюжину яиц.Когда она идет домой, все яйца, кроме трех, разбиваются. Сколько яиц осталось целыми?

    Три

  • Что треугольник сказал кругу?

    Вы бессмысленны.

  • Яйца по 0,12 доллара за дюжину. Сколько яиц можно получить за доллар?

    100 яиц, по пенни за каждое

  • Что тяжелее: тонна кирпичей или тонна перьев?

    Ни то, ни другое – они оба весят тонну.

  • Если есть три яблока и вы убираете два, сколько у вас яблок?

    У вас два яблока.

  • Сколько месяцев в году 28 дней?

    Все, потому что в каждом месяце не менее 28 дней.

  • Когда дела идут не так, на что вы всегда можете рассчитывать?

    Пальцы

  • Вы садитесь завтракать и понимаете, что у вас осталось 4 рогалика. Вы знаете, что через четыре дня у вас закончатся деньги, поэтому вы сокращаете их вдвое. Сколько у вас сейчас рогаликов?

    4 рогалика

  • Если вы умножите это число на любое другое, ответ всегда будет одинаковым.Что это за номер?

    Ноль

  • Что одна книга по математике сказала другой книге по математике?

    Вы хотите услышать мои проблемы?

  • Как футбольный болельщик перед игрой узнал, что счет будет 0: 0?

    Перед игрой всегда счет 0: 0.

  • У мужчины 9 детей. Половина из них – мальчики. Как это возможно?

    Они все мальчики.

  • В маленьком странном городке был необычный небольшой ручей с необычными рыбками в необычной маленькой команде.К местному рыбаку подошел незнакомец и спросил его, сколько весит его странная рыбка. Странный человечек ответил: «Вся рыба в этом ручье весит ровно 1/2 фунта плюс 1/2 рыбы». Разве это не странно? Сколько фунтов весит странная рыбка?

    Один

  • Сколько сторон у круга?

    Два, внутри и снаружи

  • Если есть четыре овцы, две собаки и один пастух, сколько там ног?

    Два, потому что у овец есть копыта, у собак есть лапы, только у людей есть ноги.

  • Мальчик надувает 18 пузырей,
    Потом выскакивает 6, ест 7,
    А потом выскакивает 5 и дует 1.
    Сколько осталось?

    1

  • Почему десятифунтовые банкноты 1999 года стоят больше десятифунтовых банкнот 1993 года?

    Потому что их больше

  • В одном году 365 дней. Сколько секунд в году?

    12 (второй день каждого месяца)

  • У вас корзина с 20 яблоками. У вас 20 голодных детей. Каждый ребенок должен получить яблоко, но в корзине должно остаться одно яблоко.Как ты это делаешь?

    Вы даете 19 детям по одному яблоку, а последнему отдаете корзину с последним яблоком.

  • Как разделить 10 яблок на 11 человек?

    Вы делаете яблочное пюре.

Числовые загадки для взрослых

Помните уроки математики в старшей школе? Вы, вероятно, любите математику или ненавидите ее. Цифры могут сбивать с толку, но они также могут быть забавными. Наши «Загадки с числами для взрослых» используют множество основных математических фактов.Просто не торопитесь и хорошо подумайте, и вы сможете ответить им и получить от этого удовольствие.

  • Бабушка, две матери и две дочери вместе пошли на бейсбольный матч и купили по одному билету. Сколько всего билетов они купили?

    3 билета (Бабушка тоже мать, а мать тоже дочь.)

  • Во время доставки Джейкоб может поместить 10 маленьких коробок или 8 больших коробок в картонную коробку. Всего за одну партию было отправлено 96 коробок.Маленьких ящиков было меньше, чем больших. Какое общее количество коробок он отправил?

    11 коробок
    4 маленьких коробки (410 = 40 коробок) 7 больших коробок (78 = 56 коробок)
    Итак, Джейкоб отправил 96 коробок и 11 коробок всего

  • Когда Майклу было 6 лет, его младшая сестра Лора была половина – это возраст. Если Майклу сегодня 40 лет, сколько лет Лауре?

    Ей 37 лет.

  • Что можно поставить между 7 и 8, чтобы результат был больше семи, но меньше восьми?

    Десятичный.7,8 больше 7, но меньше 8

  • Джона попросили нарисовать номер квартиры на табличках со 100 квартирами, что означает, что ему придется нарисовать числа от 1 до 100. Можете ли вы подсчитать, сколько раз ему придется раскрасить цифру 8?

    20 раз. (8, 18, 28, 38, 48, 58, 68, 78, 80, 81, 82, 83, 84, 85, 86, 87, 88, 89, 98)

  • Какое максимально возможное количество раз вы можете вычесть цифру 5 из числа 25?

    Только один раз. Это потому, что когда вы вычитаете 5 из 25 в первый раз, получается число 20, затем 15 и так далее.

  • Леонард работает в аквариуме. Когда он пытается поместить каждую черепаху в отдельный резервуар, у него на одну черепаху слишком много. Но если он ставит две черепахи на танк, у него на один танк слишком много. Сколько черепах и сколько танков у Леонарда?

    У него 3 танка и 4 черепахи

  • 300-футовый поезд, который движется со скоростью 300 футов в минуту, должен пройти через 300-футовый туннель. Сколько времени займет поездка поезда по туннелю?

    Две минуты. У передней части поезда уходит одна минута, а остальной части поезда потребуется две минуты, чтобы очистить туннель.

  • У Марлы четыре дочери, и у каждой из дочерей есть брат. Сколько детей у Марлы?

    Пять – у каждой дочери один и тот же брат.

  • Два отца и два сына в машине, а в машине всего три человека. Как?

    Это дед, отец и сын.

  • Позавчера мне был 21 год, а в следующем году – 24. Когда у меня день рождения?

    31 декабря; сегодня 1 января.

  • Мужчина описывает своих дочерей, говоря: «Все они блондинки, но двое; все брюнетки, кроме двух; и все рыжеволосые, кроме двоих.«Сколько у него дочерей?

    Трое – блондинка, брюнетка и рыжая

  • У девушки столько братьев, сколько сестер, но у каждого брата братьев вдвое меньше, чем сестер. Сколько братьев и сестер в семье?

    Четыре сестры и три брата

  • В новом магазине одежды используется уникальный метод ценообразования. Жилет стоит 20 долларов, галстук – 15 долларов, блузка – 30 долларов, нижнее белье – 45 долларов. Сколько будут стоить брюки?

    25 долларов США. Метод ценообразования взимает 5 долларов за каждую букву, необходимую для написания элемента.

  • Что ноль сказал восьмерке?

    «Хороший пояс!»

  • Какие 3 числа дают одинаковый результат при умножении и сложении?

    1, 2 и 3 (1 + 2 + 3 = 6 и 1 x 2 x 3 = 6).

  • У миссис Грин 5 дочерей. У каждой из этих дочерей есть брат. Сколько детей у миссис Грин?

    У нее 6 детей. У каждой дочери есть брат. Есть 5 дочерей и 1 сын.

Числовые загадки и ответы

А теперь давайте посмотрим, насколько хорошо вы решаете числовые загадки.Сначала мы перечислили загадки, а затем пронумерованные ответы. Также проверьте свою семью и друзей.

  1. Я трехзначный номер. Моя цифра десятков на шесть больше, чем моя цифра единиц. Моя цифра сотен на восемь меньше, чем моя цифра десятков. Какой я номер?

    193

  2. Если семь человек встретятся друг с другом и каждый пожмет друг другу руки только один раз, сколько рукопожатий будет?

    Двадцать один

  3. В полночь идет дождь, но прогноз на завтра и на следующий день ясен.Будет солнечная погода через 48 часов?

    Нет, не будет солнечно, потому что на улице будет темно. Через 48 часов снова будет полночь.

  4. Два отца и 2 сына провели день на рыбалке, но поймали только 3 рыбы. Этого хватило, чтобы каждому из них хватило по одной рыбке. Как это возможно?

    На рыбалке было всего 3 человека. Был один отец, его сын и сын его сына. Это означает, что было 2 отца и 2 сына, так как один из них – отец и сын.

  5. Слово, которое я знаю, шесть букв в нем, удалите одну букву, а осталось 12, что это?

    Десятки

  6. 81 x 9 = 801.Что вам нужно сделать, чтобы это уравнение стало верным?

    Переверните вверх дном. 108 = 6 x 18

  7. Лошадь была привязана к веревке длиной 5 метров, а корм для лошади находился в 15 метрах от лошади. Как лошадь добралась до еды?

    Веревка ни к чему не привязана, чтобы он мог достать еду.

  8. Вы водитель автобуса на вокзале. 10 садятся в автобус, и никто не выходит. Вы проезжаете 10 миль до другой станции, где 3 человека входят и 1 выходит. Вы проезжаете еще 10 миль только для того, чтобы собрать 25 человек и уехать 5 человек.Затем вы проезжаете 3 мили, а из автобуса выходят 12 человек. Вы едете обратно к месту, зная, что по пути будет еще пара остановок, пока поездка не закончится. На предпоследней станции 5 человек садятся и 2 выходят. А на последней станции (5,5 миль) без причины садятся 25 человек и сходит 1. Какого цвета глаза у водителя автобуса?

    Вы управляете автобусом, поэтому цвет может быть любого цвета ваших глаз.

  9. Если пятнадцать ворон на заборе и фермер стреляет треть из них, сколько ворон останется?

    Нет.Остальные вороны улетели, когда услышали выстрел.

  10. На столе лежат три спички. Без добавления еще одной марки на три соответствует четырем. Вам не разрешается прерывать какие-либо матчи.

    Сформируйте из трех спичек римскую цифру четыре.

  11. У Тулии 29 долларов. Она купила 4 книжки-раскраски по 3 доллара каждая и 4 коробки цветных карандашей по 2 доллара каждая. Остальные деньги она потратила на маркеры. Сколько денег она потратила на маркеры?

    Она потратила 9 долларов.00 на маркерах

  12. Вы и еще четыре человека ограбили банк на 200 000 000 долларов. Вы поровну делите деньги. На каждого человека вы получаете 25% от суммы, которую вы получаете.

    Вы получаете 100% сокращение, потому что 25% x 4 = 100%.

  13. В зоопарке 100 пар собак. На каждую собаку рождается по две пары малышей. К сожалению, 23 собаки не выжили. Сколько всего собак останется?

    977 собак (100 x 2 = 200; 200 + 800 = 1000; 1000 – 23 = 977)

  14. Группа студентов стоит под палящим солнцем лицом к западу во время марша.Вождь крикнул им: Поверните направо! О очередь! Налево! В конце этих команд, в каком направлении смотрят ученики?

    Восток. Они поворачиваются на 90 градусов при повороте вправо, и они поворачиваются на 180 градусов при повороте вокруг, и, наконец, они поворачиваются на 90 градусов при повороте влево. Поэтому теперь ученики смотрят на восток.

  15. Мужчина вдвое старше своей младшей сестры. Ему тоже вдвое меньше, чем их папе. Через 50 лет возраст сестры станет вдвое меньше возраста их отца.Какого возраста сейчас мужчина?

    Ему 50 лет.

  16. Удвойте и умножьте на 4. Затем разделите на 8, и вы получите еще раз. Какой это номер?

    Любое число. Если число удваивается и умножается на 4, оно фактически умножается на 8, поэтому деление на 8 снова вернет вам исходное число.

  17. В Бартлсвилле есть магазин одежды. Владелец придумал собственный метод ценообразования на товары. Жилет стоит 20 долларов, носки – 25 долларов, галстук – 15 долларов, блузка – 30 долларов.Сколько будет стоить пара нижнего белья, используя этот метод?

    45 долларов США. Метод ценообразования состоит из взимания 5 долларов США за каждую букву, необходимую для написания элемента.

  18. Я трехзначный номер. Моя вторая цифра в 4 раза больше третьей. Моя первая цифра на 3 меньше, чем вторая. Кто я?

    141

  19. Я добавляю пять к девяти и получаю два. Ответ правильный, но как?

    Когда будет 9 утра, прибавьте к нему 5 часов, и вы получите 14:00.

  20. Используя только сложение, как можно сложить восемь восьмерок, чтобы получить число 1000?

    888 + 88 + 8 + 8 + 8 = 1000

Итак, на этом наш сборник загадок с числами заканчивается.Мы надеемся, что наш выбор оказался интересным и интересным. Поделитесь ими с друзьями, семьей, коллегами и учениками в классе. Хотя некоторые из них окажутся сложными, даже самый молодой и наименее опытный решатель числовых загадок повысит свои способности к разгадыванию загадок. Помните: «практика ведет к совершенству». Удачи и загадки!

Сьюзен изучала английский язык с двойным вторым по специальности «гуманитарные науки и бизнес» в Университете штата Аризона и получила степень магистра управления образованием в Университете Либерти.Она преподавала с четвертого по двенадцатый классы как в государственных, так и в частных школах. Предметы включали английский язык, американскую и всемирную историю и географию, математику, землю и физические науки, Библию, информационные технологии и творческое письмо.

Сьюзен писала-фрилансером более десяти лет, за это время она написала и отредактировала книги, газетные статьи, биографии, книжные обзоры, руководства, описания районов для риэлторов, презентации Power Point, резюме и множество других проектов.

Прочитать полную биографию

40 загадок о числах – РЕШИТЬ или УМЕРЬ

logicmathtricky

Сколько раз вы можете вычесть 5 из 25?

Всего один раз, потому что после того, как вы вычтете что-нибудь из него, это больше не 25.

Покажи мне ответ Поделись загадкойcleansimplemath

В нем есть слово и шесть букв. Уберите одну, и останется двенадцать. Что это за слово?

Десятки.

Подскажите ответ Подскажите загадку логикоматика

Если, Фернандо + Алонсо + Макларен = 6 Фернандо х Алонсо = 2 Алонсо x Макларен = 6 Потом, Макларен х Фернандо =?

3 или 0.2-6 Алонсо + 8 = 0 (Алонсо – 4) (Алонсо – 2) = 0 Следовательно; Алонсо = 4 или 2 Примем значение Алонсо равным 2. Фернандо = 2/2 = 1 Макларен = 6/2 = 3 Следовательно; Макларен х Фернандо = 3 х 1 = 3 Примем значение Алонсо равным 4. Фернандо = 2/4 = 0,5 Макларен = 6/4 = 1,5 Следовательно; Макларен x Фернандо = 1,5 x 0,5 = 0,75

Покажите мне ответ Поделиться загадкой logicmath

Какое наименьшее число может быть выражено как сумма кубиков двух различных наборов чисел?

Харди-Рамануджан открыл 1729 год как магическое число.3 = 1729 Номер такси Ta (2)

Подскажите ответПоделиться загадкойlogicmathsimpleclean

Половина двух плюс два равна двум или трем?

Три. Кажется, что это может быть почти то же самое, но если следовать математическому порядку действий, деление выполняется перед сложением. Итак … половина из двух равна одному. Затем добавьте два, и ответ – три.

Покажите мне ответПоделиться загадкой

99 /.99 или же 99 + 9/9

Покажите мне ответПоделиться загадкойcleanlogicmathsimple

Создайте число, используя только цифры 4,4,3,3,2,2,1 и 1. Так что у меня может быть только восемь цифр. Вы должны убедиться, что единицы разделены одной цифрой, двойки разделены двумя цифрами, тройки разделены тремя цифрами, а четверки разделены четырьмя цифрами.

41312432.

Покажите мне ответПоделитесь загадкойлогикматематикапростая чистота

Я знаю число, которое при умножении на 9 i.e 9 18 27 36 45 … Вывод состоит из числа, содержащего только одну цифру. Вы можете определить номер?

12345679 12345679 × 9 = 111111111 (только единицы) 12345679 × 18 = 222222222 (только 2 с) 12345679 × 27 = 333333333 (только 3 с) 12345679 × 36 = 444444444 (только 4 с) 12345679 × 45 = 555555555 (только 5 с)

Покажи мне ответ Поделись загадкой логика

Примечание: эту загадку нужно разгадывать ТОЛЬКО В ВАШЕЙ ГОЛОВЕ, а НЕ с помощью бумаги и ручки. Возьмите 1000 и прибавьте к нему 40.Теперь добавьте еще 1000. Теперь добавьте 30. Еще 1000. Теперь добавьте 20. Теперь добавьте еще 1000. Теперь добавьте 10. Что в итоге?

Ответ 4100, проверьте это на калькуляторе. Вы думали, 5000? Большинство людей ошибочно добавляют 100 как 1000.

Покажите мне ответПоделиться загадкойлогикоматематикапросто

Все мы знаем, что квадратный корень из числа 121 равен 11. Но знаете ли вы, что такое квадратный корень из числа «12345678987654321»?

111111111 Объяснение: Это математический магический ряд квадратного корня как: Корень квадратный из числа 121 равен 11 Корень квадратный из числа 12321 равен 111 Корень квадратный из числа 1234321 1111 Корень квадратный из числа 123454321 11111 Корень квадратный из числа 12345654321 111111 Корень квадратный из числа 1234567654321 равен 1111111 Корень квадратный из числа 123456787654321 11111111 Квадратный корень из числа 12345678987654321 равно 111111111 (ответ)

Покажите мне ответ Поделитесь загадкой

30 загадок по математике для детей (разумеется, с ответами)

С помощью этих математических загадок мы немного повеселимся в предмете, который некоторые люди считают несколько сухим.

Математика – это предмет, по которому многим детям требуется дополнительная помощь или дополнительные занятия. Тема может быть сложной. В большинстве случаев дети учатся тому, чему их никогда не учили. Это помогает выбрать легкий подход вместо процесса «пробурить и убить», который, так сказать, сводит на нет удовольствие от уравнения. 🙂

Эти математические словесные головоломки пробуждают нестандартное мышление и повороты, чтобы пробудить любопытство и интерес к математике. Надеемся, они вам понравятся.

Загадки на хорошую математику для детей

1.) Если на постройку сарая ушло 8 человек по 10 часов, сколько времени потребуется 14 людям, чтобы построить такой же сарай?

2.) Ферма Тома продает яйца по 12 центов за дюжину. Сколько яиц можно получить за один доллар?

3.) Если бы вы перемножили все числа на цифровой клавиатуре смартфона, какова была бы сумма?

ОТВЕТОВ: 1.) Нет – сарай уже построен! 2.) 100 яиц. 12 центов за дюжину означает, что яйца продаются по пенни за штуку. 3.) Ноль. Поскольку числа от 0 до 9 представлены на панели набора номера, любое значение, умноженное на ноль, равно нулю!

Загадки по математике с ответами

4.) Разделите 40 пополам, а затем прибавьте 5. Каков ответ?

5.) Можете ли вы сложить количество сторон треугольника, пятиугольника и шестиугольника? Сколько всего сторон?

6.) Сэму 14 лет, а Бритте вдвое меньше. Сейчас Сэму 34 года. Сколько лет Бритте?

ОТВЕТОВ: 4.) 85, деление пополам фактически удваивает число! 5.) 14 сторон. 6.) 27 лет. (Вы ведь не вдвое моложе ее, не так ли?)

Детям понравятся эти веселые загадки о географии.

Веселые загадки на математику

7.) Что весит больше: фунт железа или фунт перьев?

8.) Вы хотите купить закуску в автомате за 75 центов. У вас есть четверть, 2 цента, 4 цента и 5 пенни. У вас достаточно денег, чтобы купить закуску?

9.) Джон работает в зоопарке. Если он попытается дать каждому пингвину отдельную выставку, то на одного пингвина у него будет слишком много. Если он выставит двух пингвинов на каждую выставку, то у него на одну выставку будет слишком много.Сколько здесь экспонатов? Сколько пингвинов?

ОТВЕТОВ: 7.) Оба будут одинаковыми. Фунт – это фунт независимо от объекта. 8.) Нет, у вас всего 70 центов. 9.) 3 экспоната, 4 пингвина.

Вам может понравиться Все о LEGO – История, часто задаваемые вопросы и забавные факты

Математические головоломки

10.) Если вы умножите меня на любое другое число, ответ всегда будет таким же. Какой я номер?

11.) Можете ли вы сделать число семь даже без добавления, вычитания, деления или умножения на любые другие числа?

12.) Две матери и две дочери ходят за покупками в торговый центр. Каждый покупает предмет одежды, но они приходят домой только с тремя предметами одежды. Это почему?

ОТВЕТОВ: 10.) Ноль. 11.) Да, избавьтесь от «s». Семерка становится четной. 12.) Группа состоит из бабушки, матери и дочери.

Сложные математические задачи со словами

13.) Три куры могут отложить три яйца за три минуты? Сколько яиц откладывают куры за 2 часа при стабильной яйценоскости?

14.) Джош подбрасывает монету пять раз, и каждый раз она выпадает орлом. Каковы шансы, что ему выпадет решка при следующем броске?

15.) У вас есть 5 разных пар носков. Две пары носков – зеленые, одна пара – красная, одна пара – синяя и одна пара – желтая. Каковы ваши шансы случайно выбрать пару зеленых носков для ношения?

ОТВЕТОВ: 13.) 120 яиц. 14.) 50% – шансы не меняются в зависимости от предыдущих результатов по подбрасыванию монеты. 15.) Два из пяти или 40%.

Сложные математические загадки с ответами

16.) Если двое – компания, а трое – толпа, то какие четыре и пять?

17.) Я никогда не являюсь целым числом. Если разделить число на меня, то число возрастет. Какой я тип номера?

18.) Вам на ужин дают большую пиццу пепперони, и вам нужно разделить ее на 8 ломтиков для семейного ужина. Сколько раз нужно разрезать пиццу, чтобы разделить ее на восемь частей?

ОТВЕТОВ: 16.) Девять. 17.) Дробь. 18.) Четыре раза

Головоломки по математической логике

19.) У Джоша 5 сыновей, и у каждого из них есть сестра. Сколько детей у Джоша?

20.) Когда два числа складываются, их сумма равна 46. Одно из чисел на 6 больше другого. Какие два числа?

21.) Полицейский остановил вашего старшего брата за превышение скорости в зоне со скоростью 35 миль в час. Офицер дает ему билет на 55 долларов и говорит, что за каждый билет взимается штраф в размере 10 долларов в дополнение к дополнительным 5 долларам за каждую 1 милю в час сверх установленной скорости.Как быстро продвигался твой брат?

ОТВЕТОВ: 19.) Шесть – у всех сыновей одна сестра! 20.) 20 и 26. 21.) 44 мили в час.

Лучшие математические загадки

22.) В семье из 8 сестер, каждая из которых родилась с разницей в 3 года, младшей сестре 12 лет. Сколько лет старшей сестре?

23.) Сумма двух последовательных чисел равна 37. Что это за два числа?

24.) Число 5 преобразуется в 11. Число 7 преобразуется в 15. И число 9 преобразуется в 19.Что происходит с числом 12?

ОТВЕТОВ: 22.) 33 года. 23.) 18 и 19. 24.) Преобразуется в 25. Формула: умножить на 2, а затем на +1.

Стамперы

25.) Какой следующий номер в последовательности? 2, 4, 8, 16…

26.) 1 = 3, 2 = 3, 3 = 5, 4 = 4, 5 = 4, 6 = 3, 7 = 5, 8 = 5, 9 = 4, 10 =? Сможете ли вы завершить последовательность?

27.) Два четных числа, сложенные вместе, всегда дают четное число. Добавление четного и нечетного числа всегда дает нечетное число.Какой тип числа всегда получается при сложении двух нечетных чисел?

ОТВЕТОВ: 25.) 32 – формула удваивает предыдущее число. 26.) 10 = 3. Число после знака равенства относится к количеству букв в числе перед знаком равенства. 27.) Четное число!

Детские загадки на математику

28.) Сколько времени в четверти часа?

29.) У вас есть 24,60 доллара с 7 купюрами и 3 монетами. Какие купюры и монеты у вас есть?

30.) Если бы вы написали случайное целое число от 1 до 10, то каковы шансы, что это число будет нечетным?

ОТВЕТОВ: 28.) 15 минут 29.) 30.) Один $ 10, два $ 5, четыре $ 1, 2 четверти и один цент. 50%.

Ключ для подсчета очков для нашего конкурса загадок по математике

Если вы правильно ответили 24–30: Студент – Молодец!

Если вы правильно ответили 17–23: B Студент – Хорошая работа!

Если вы ответили <17 правильных ответов: Требуется улучшение - Практикуйтесь больше.

– Грег Джонсон, Кейси О’Халлоран и Майкл О’Халлоран

Грег и Майк являются соучредителями ListCaboodle.com, а Кейси – наш эксперт по викторинам.

Энкор

Вы находитесь на странице математических загадок.

Вам понравится:

Головоломки

Загадки науки

Викторина “Знай свои идиомы”

Загадки известных женщин

Викторина по черной истории

10 загадок по математике с ответами

15 января 2021 г.

Время чтения: 8 минут

Введение

Математические загадки с ответами помогают детям развивать интерес и способности к математике.Родители могут помочь своим детям отгадать эти загадки, представленные в статье.

Математические загадки могут быть сложной задачей, требующей сложных решений. Эти прикладные вопросы помогут вашему ребенку мыслить нестандартно. Они помогут вашим детям улучшить свой IQ и успеваемость.

Попросите вашего ребенка разгадывать математические загадки и головоломки, представленные в этой статье, чтобы вовлечь его в полезные для учебы занятия.

Также читайте:


Загадки с ответами

Вот PDF-файл, который состоит из еще нескольких забавных и сложных математических загадок, которые вам понравится читать и отвечать на них.Они могут думать об этих математических загадках с ответами как о разминке, которая готовит молодых учеников к предстоящим задачам. Нажмите кнопку загрузки, чтобы узнать больше.

📥 15 забавных и сложных математических загадок

Загрузить


10 сложных математических загадок с ответами для детей

1. В картонной коробке находится несколько яблок, которые были разделены на две равные части и проданы 2 торговцам Таруну и Танмаю.У Таруна было два фруктовых магазина, и он решил продавать одинаковое количество яблок в обоих магазинах A и B соответственно. Таня зашла в магазин А и купила детям все яблоки. Но одно яблоко осталось после того, как разделили все яблоки между ее детьми. Каждому ребенку досталось одно яблоко, найти минимальное количество яблок в коробке?

Пояснение:
Пусть количество детей будет X
Следовательно, количество купленных Таней яблок = (X + 1)
Общее количество яблок в магазине A = (X + 1)
Общее количество яблок, купленных Торговцем Таруном = 2 (X + 1)
Общее количество яблок в коробке = 4 (X + 1)
Сейчас минимальное количество яблок в коробке:
Предположим:
X не равно 0
Х не равно 1 (у Тани нет ни одного ребенка)
X равно 2 (у Тани минимум 2 ребенка)
Когда X = 2
4 (Х + 1)
= 4 (2 + 1)
= 12
Таким образом, минимум 12 яблок

2.Обезьяна пытается взобраться на кокосовую пальму. Он делает 3 шага вперед и скользит на 2 шага вниз. Каждый шаг вперед составляет 30 см, а каждый шаг назад – 40 см. Сколько ступенек нужно, чтобы подняться на 100-сантиметровое дерево?

Пояснение:
Каждый раз, когда Обезьяна пытается подняться:
Поднимается вверх = 3 X 30 см = 90 см (занимает 3 шага)
Скольжение вниз = 2 X 40 см = 80 см (2 шага)
Общее количество шагов, сделанных при каждой попытке перемотки = 3 шага + 2 шага = 5 шагов
Эффективно перемещается на 10 см вверх (90 см – 80 см = 10 см)
100 см / 10 см = 10
Следовательно, обезьяне необходимо преодолеть 10 X 5 шагов = 50 шагов, чтобы достичь вершины

.

3.На строительной площадке требуется как минимум 5 рабочих, чтобы выполнить работу за 12 дней. Сколько дней потребуется, если на одну и ту же работу будут наняты 12 рабочих?

Пояснение:
Пусть X будет объем работы, которую необходимо сделать
Пусть Y будет количеством дней, затраченных 12 рабочими
Уравнение работы 5 рабочих:
X / 5 = 12 дней
Х = 60
Соотношение работы 12 рабочих:
X / 12 = Y
60/12 = Y
Y = 5

4.У вас есть фунт хлопка и фунт железа, что будет больше?

Пояснение:
1 фунт хлопка = 1 фунт железа
Фунт – это единица веса.

5. Сумма возраста дочери и матери составляет 55 лет. Возраст дочери – это возраст матери, обратный возрасту матери. Найдите возраст матери и дочери, если возраст матери больше 40 лет.

Матери 41 год, дочери 14 лет

Пояснение:
Пусть первая цифра возраста матери будет X
Пусть вторая цифра возраста матери будет Y
. Таким образом, возраст матери – XY
Возраст дочери –
YX Допустим, X> Y, так как мать всегда старше дочери
Сумма возраста определяется уравнением:
((10X) + Y) + ((10Y) + X) = 55 90 · 108 11X + 11Y = 55
Следовательно, возможный возраст может быть:
Мать: 32 года и дочь: 23
Мать: 41 год и дочь: 14
Как упоминалось в загадке, матери больше 40 лет, поэтому мы выбираем второе решение.

6. У вас 50 печенья. Сколько раз вы можете вычесть 5 из 50 печенья?

Пояснение:
Один раз только потому, что после этого не будет 50 печенья.
Если вычесть 5 печений, получится 45, а не 50.

7. Получить четное число из 7 без сложения, вычитания, умножения или деления любого числа?

Пояснение:
СЕМЬ – S = ЧЕТНОЕ

8.Сейчас 9 утра, Рита 2 часа учится, 1 час принимает ванну, потом 1 час обедает. Сколько часов осталось до 9 утра завтра?

Пояснение:
Рите требуется 4 часа (2 + 1 + 1) на учебу, купание и прием пищи.
С 9:00 сегодня до 9:00 завтра круглосуточно.
24 часа – 4 часа = 20 часов

9. Есть 8 яблок, забираешь 4 и возвращаешь 1 яблоко. Сколько у вас яблок?

Пояснение:
Вы купили себе 4 яблока и вернули 1 яблоко
Следовательно, 4-1 = 3 яблока

10.Скорость поезда составляет 3 м / сек, а пересечение фонарного столба занимает 10 секунд. Какая длина поезда?

Пояснение:
Поезд преодолевает 3 м за 1 секунду
Следовательно, за 10 секунд он покроет = 3 м X 10 секунд = 30 м
Через 10 секунд поезд длиной 30 м пересечет фонарный столб.

Вы можете найти больше математических загадок и головоломок для своего ребенка на Cuemath. Это платформа для детей, которая предлагает индивидуальные математические загадки, головоломки, симуляторы и игры, чтобы помочь развить их математические способности, а также улучшить их успеваемость.

Все основные математические задачи вашего ребенка будут решены лучшими учителями математики страны. Запишитесь сегодня, чтобы начать путешествие своего ребенка по математике.


Заключение

Решите эти сложные математические загадки вместе с ребенком, чтобы понять их концептуальную ясность. Если ваш ребенок плохо успевает по математике, вы можете выяснить его недостатки, проанализировав его подход к решению этих загадок.

Сядьте вместе с ребенком, разгадывая математические загадки, чтобы оценить способность решать проблемы.Делитесь подсказками, которые помогут им правильно разгадать большинство загадок, и объясните каждый ответ, чтобы убедиться, что ваш ребенок никогда не повторяет похожие ошибки при решении задач.


О компании Cuemath

Cuemath, удобная для учащихся платформа математики и кодирования, проводит регулярные онлайн-классы для учебы и развития навыков, а их приложение Mental Math для iOS и Android представляет собой универсальное решение для детей, развивающее несколько навыков. структуру Cuemath Fee и подпишитесь на бесплатную пробную версию.


Внешние ссылки

Для более сложных математических головоломок воспользуйтесь приведенными ниже ссылками с других веб-сайтов:

‘7 – это число, которое начинается с буквы’ загадка, которая сбивает TikTok с толку! Вот ответ

Загадки и головоломки стали чрезвычайно популярными среди людей, которые оставались дома во время карантина. Но ведь это не весело, когда не можешь разгадать загадку, правда?

Период карантина вдохновил многих людей на то, чтобы держать свой мозг активным и разгадывать ряд загадок.

В то время как некоторые твердо выбрали комфорт дивана и Netlfix, другие нашли новые способы занять себя и окружающих их людей и развлечь их.

С тех пор в социальных сетях распространилась интересная загадка, которая сбила с толку многих пользователей.

Вот что мы узнали о загадке под названием «7 – это число, начинающееся с буквы».

Andia / Universal Images Group через Getty Images

Загадка: В чем вопрос?

Загадка звучит так:

  • 7 – единственное число, которое начинается с буквы.

Итак, почему это так? Все числа начинаются с буквы – у вас есть один, два, три, четыре, пять и т. Д.

Хотя загадки и головоломки обычно доставляют удовольствие, некоторые из них могут иметь не столь очевидные объяснения.

Ответ на разгадку загадки

По словам людей в Интернете, ответ заключается в написании чисел.

Например, если вы объедините первый символ этих чисел – 1, 2, 3, 4, 5, 6, 8, 9 – и остальные буквы, вы получите 1ne, 2wo, 3hree, 4our, 5ive, 6ix. , 8ight, 9ine.

Однако, когда вы пишете семь как «7even», число не звучит как семь – оно больше похоже на слово «даже».

Понял? Что ж, не будем лгать, мы запутались так же, как и вы, но это объяснение, которое придумали пользователи социальных сетей.

Этот контент не может быть загружен

кто-то сказал, что это из-за этого:
1ne 2wo 3hree 4our 5ive 6ix seven 8ight 9ine

– val⁷ ☆ (@joonsrare) 25 июня 2020 г.

Если вам нужно более подробное объяснение, кто-то создал на YouTube целый видеоролик о загадке.

Вы можете посмотреть это внизу, хотя загадка все еще не имеет для нас смысла.

Из других новостей, кто парень Ани Тейлор-Джой? Изучена свидательная жизнь ведущего SNL!

31 сложная математическая загадка и словесная задача для молодых Эйнштейнов

Никого не оставит равнодушным длинная математическая задача со словом, начальные строки которой – . 67 человек в поезде, который едет на запад со скоростью 45 миль в час… – склонны к решению. либо волнуют нас, либо наполняют холодным ужасом.В то время как все мы без усилий ежедневно сталкиваемся с простой арифметикой, математические загадки и словесные задачи, как правило, останавливают нас: они представляют собой тернистую, восхитительную комбинацию абстрактного и реального мира – вот почему они идеально подходят для Дети.

Чтобы решить математическую загадку, дети должны преобразовать всего несколько предложений о реальном сценарии в правильную комбинацию математических уравнений, используя логику, творческое решение проблем и здравый смысл для взлома кода. Математические загадки, особенно самые сложные, представляют собой приятную задачу для детей (и, будем откровенны, взрослых).

Хорошая новость заключается в том, что сокрытие сложного уравнения за веселой и увлекательной загадкой – лучший способ заинтересовать вашего ребенка математикой, прежде чем он столкнется с большим разделением квадратичной формулы в восьмом классе. Эти сложные математические загадки развлекают детей, постепенно оттачивая их логические и математические навыки. Скорее всего, ты тоже чему-нибудь научишься.

Загадка: Во время доставки Том может поместить 10 маленьких коробок или 8 больших коробок в картонную коробку. Всего за одну партию было отправлено 96 коробок.Маленьких ящиков было меньше, чем больших. Какое общее количество коробок он отправил?
Ответ: 11 коробок ( Пояснение: 4 маленьких коробки (4 10 = 40 коробок) + 7 больших коробок (7 8 = 56 коробок). Итак, 96 коробок и всего 11 коробок.)

Загадка : Когда Мигелю было 6 лет, его младшей сестре Лейле исполнилось полгода. Если Мигелю сегодня 40 лет, сколько лет Лейле?
Ответ: Ей 37 лет.

Загадка: Вам дано 3 положительных числа.Вы можете сложить эти числа и умножить их вместе. Результат будет таким же. Какие числа?
Ответ: 1, 2 и 3

Загадка: Если полторы курицы отложат полтора яйца за полтора дня, сколько яиц отложит полдюжины куриц за полдюжины дней. ? Ответ: 2 дюжины.

Загадка: Что можно поставить между 7 и 8, чтобы результат был больше семи, но меньше восьмерки?
Ответ: Десятичное число.7,8 больше 7, но меньше 8

Загадка: Тома попросили нарисовать числа за пределами 100 квартир, что означает, что ему придется раскрасить числа от 1 до 100. Можете ли вы подсчитать, сколько раз ему придется раскрасить число 8?
Ответ: 20 раз (8, 18, 28, 38, 48, 58, 68, 78, 80, 81, 82, 83, 84, 85, 86, 87, 88, 89, 98)

Загадка: Какое максимальное количество раз можно вычесть число 5 из 25?
Ответ: Только один раз.Это потому, что когда вы вычитаете 5 в первый раз, получается число 20, затем 15 и так далее.

Загадка: Что весит больше 16 унций соды или фунта чистого золота?
Ответ: Ни то, ни другое. Оба они весят одинаково!

Загадка: Леон работает в аквариуме. Когда он пытается поместить каждую черепаху в отдельный резервуар, у него на одну черепаху слишком много. Но если он ставит двух черепах на танк, у него на танке слишком много. Сколько черепах и сколько танков у Леона?
Ответ: У него 3 танка и 4 черепахи.

Загадка: Полная стоимость пары туфель и толстовки составляет 150 долларов. Толстовка с капюшоном стоит на 100 долларов больше, чем пара обуви. Сколько стоит каждый предмет?
Ответ: Толстовка стоит 125 долларов, обувь – 25 долларов

Загадка: Если есть 4 яблока и вы забираете 3, сколько у вас есть?
Ответ: Вы взяли 3 яблока, значит, у вас 3 яблока!

Загадка: 300-футовый поезд движется со скоростью 300 футов в минуту, должен пройти 300 футов.длинный туннель. Сколько времени займет поездка поезда по туннелю?
Ответ: Две минуты. У передней части поезда уходит одна минута, а остальной части поезда потребуется две минуты, чтобы очистить туннель.

Загадка: Мобильный телефон и чехол для телефона в сумме стоят 110 долларов. Сотовый телефон стоит на 100 долларов дороже чехла для телефона. Сколько стоил мобильный телефон?
Ответ: 105 долларов (не 110 долларов)

Загадка: Роберт и Дэвид сыграли несколько матчей в гольф друг против друга за неделю.На каждом матче играли за пиццу, но до конца недели пиццы не покупали. Если когда-либо Роберт и Дэвид выигрывали одинаковое количество побед, эти пиццы отменялись. Роберт выиграл четыре матча (но без пиццы), а Дэвид выиграл три пиццы. Сколько раундов в гольф было сыграно?
Ответ: Одиннадцать
Пояснение: Дэвид выиграл 7 матчей, 4 отменил 4 победы Роберта и еще 3 выиграл пиццу.

Загадка: Я трехзначное число.Моя вторая цифра в 4 раза больше третьей. Моя первая цифра на 3 меньше, чем вторая. Кто я?
Ответ: 141

Загадка: Я прибавляю пять к девяти и получаю два. Ответ правильный, но как?
Ответ: Когда будет 9 утра, прибавьте к нему 5 часов, и вы получите 14 часов.

Загадка: В зоопарке 100 пар собак; На каждую собаку рождаются пары младенцев. К сожалению, 23 собаки не выжили. Сколько всего собак останется?
Ответ: 977 собак
Пояснение: 100 x 2 = 200; 200 +800 = 1000; 1000-23 = 977

Загадка: Группа студентов стояла под палящим солнцем лицом к западу во время марша, прошедшего мимо.Вождь крикнул им: Поверните направо! О очередь! Налево! В конце этих команд, в каком направлении сейчас смотрят ученики?
Ответ: Восток. (Пояснение: они повернутся на 90 градусов при повороте вправо, и они повернутся на 180 градусов при повороте вокруг, и, наконец, они повернут на 90 градусов при повороте влево. Таким образом, ученики теперь смотрят на восток.)

Загадка: Половина – это треть. Что это?
Ответ: 1 1/2.

Загадка: У вас есть два U.S монет общей стоимостью 0,30 доллара. Один из них не никель. Какие две монеты?
Ответ: Один – четверть, а другой – никель.

Загадка: Яйца по 0,12 доллара за дюжину. Сколько яиц можно получить за доллар?
Ответ: 100 яиц по пенни каждое.

Загадка: Утка – 9 долларов, паук – 36 долларов, пчела – 27 долларов. На основании этой информации, сколько денег будет отдано кошке?
Ответ: 18 долларов (4.50 на ногу)

Загадка: «Сколько стоит этот мешок картошки?» спросил мужчина. «32 фунта, разделенные на половину собственного веса», – сказал бакалейщик. Сколько весила картошка?
Ответ: 8 фунтов.

Загадка: Я трехзначное число. Моя цифра десятков на шесть больше, чем моя цифра единиц. Моя цифра сотен на восемь меньше, чем моя цифра десятков. Какой я номер?
Ответ: 193

Загадка: Мужчина вдвое старше своей младшей сестры.Он тоже вдвое младше своего отца. Через 50 лет возраст его сестры станет вдвое меньше возраста их отца. Какого возраста сейчас мужчина?
Ответ: 50 лет.

Загадка: Как сложить восемь четверок, чтобы в сумме получилось 500?
Ответ: 444 + 44 + 4 + 4 + 4 = 500

Загадка: Если семь человек встретятся друг с другом и пожмут друг другу руки только один раз, сколько рукопожатий будет?
Ответ: 21

Загадка: Если четыре человека могут построить четыре стола за четыре часа, сколько столов восемь человек могут построить за восемь часов?
Ответ : 16 таблиц

Загадка: Когда Лизе было 6 лет, ее сестра Люси была вдвое моложе.Если Люси сегодня 40 лет, сколько лет Люси?
Ответ: 37
Пояснение: Люси на 3 года младше Лизы.

Загадка: Если вы покупаете петуха с целью откладывания яиц и рассчитываете получать по три яйца каждый день на завтрак, сколько яиц у вас будет через три недели?
Ответ: Нет. Петухи яиц не откладывают.

Загадка: У фермера на земле 19 овец. Однажды обрушивается сильный шторм, и все, кроме семи, разбегаются.Сколько овец осталось у фермера?
Ответ: Семь. Все, кроме семи, сбежали.

30+ загадок с хитрыми числами, чтобы проверить свой ум

Лучшие числовые загадки, чтобы бросить вызов вашему мозгу?

Итак, мы составили уникальный список лучших загадок с числами, в которые вы можете играть и получать удовольствие.

Большинство записей в этом списке краткие и по существу. И поэтому идеально подходят для детей, подростков и взрослых.

Если вы ищете отличные загадки на время разгадывания загадок или что-нибудь обычное, чтобы разыграть себя или с друзьями или семьей, эти загадки с числами для вас.

Приступим.

Загадки с хитрыми числами и головоломки с ответами

1. Цифры для детей

Загадка: Сколько раз можно вычесть 2 из 8?

Ответ: Один раз!

Как только вы вычитаете в первый раз, уже не 8

2. Целые числа

Загадка: Какие только два целых числа дают двузначные числа при сложении и однозначные при умножении?

Ответ: Один и девять

3.Лоток для яиц

Загадка: Лиза дважды взяла с подноса дюжину яиц. Сколько яиц осталось?

Ответ: Шесть.

Поднос означает 30, а дюжина означает 12.

4. Логическая загадка для детей

Загадка: Как получится 14 + 2 = 4?

Ответ: Воспользуйтесь часами.

14 + 2 = 16, что соответствует 4 часам

5. Интервалы перезвона часов

Загадка: Если часы бьют 6 раз за 5 минут, сколько раз они будут звенеть за 10 минут?

Ответ: 11 раз.

Часы будут звенеть каждую минуту, начиная с одной минуты, которая составляет вторую минуту.

6. Дилемма сложения и вычитания

Загадка: Если сложить шесть и четыре, получится десять. Но если сложить шесть и десять, получится четыре. Как это возможно?

Ответ: Можно при часах

7. Оружие часов

Загадка: Сколько раз стрелки часов перекрываются за день? (Подсказка: это получит только гений)

Ответ: Двадцать два раза.

Например, в это время руки будут перекрываться:

12:00

1:05

2:11

3:16

4:22

5:27

6:33

7:38

8:44

9:49

10:55

Что касается вечера, то в это время руки будут перекрываться:

12:00

13:05

14:11

15:16

16:22

17:27

18:33

19:38

20:44

21:49

22:55

8.Разделить 10 пополам

Загадка: Что получится, если взять 10 и разделить его пополам?

Ответ: 20!

Большинство людей скажут, что это 5

9. Четные числа

Загадка: Как проще всего получить четное число из семи?

Ответ: Уберите букву S из семи, и вы останетесь даже с

10. Килограмм перьев против песка

Загадка: Если ребенок изо всех сил пытается поднять килограмм перьев.Какова вероятность того, что малыш поднимет килограмм песка?

Ответ: Очень вероятно.

Вес такой же, один килограмм

11. Яйцо в день

Загадка: Если я куплю дюжину яиц и решу съесть одно яйцо на завтрак каждый день. Как долго продержатся яйца?

Ответ: Всего 12 дней

12. Простые загадки с числами

Загадка: Как проще всего получить число 4 из ПЯТИ?

Ответ: Обрежьте первую и последнюю буквы, чтобы получить F.IV.E. IV – римское число 4

.

13. Загадки римского числа

Загадка: Любите римские числа?

Ответ: Я, например, большой поклонник

14. Римская числовая загадка

Загадка: Какие римские числа, если их критиковать в Китае, могут доставить вам неприятности?

Ответ: XI, президент

15. Девятки от 0 до 100

Загадка: Сколько девяток между 0 и 100?

Ответ: Двадцать

16.Скорость роста

Загадка: В озере Виктория есть водяной гиацинт, размер которого через день увеличивается вдвое. Если на то, чтобы покрыть все озеро, потребовалось 50 дней, сколько дней потребовалось, чтобы покрыть половину озера?

Ответ: 49 дней

17. Один полтора килограмма

Загадка: Один кирпич весит один килограмм и полкирпича. Какой вес у одного кирпича?

Ответ: Два килограмма

18.Правильное уравнение

Загадка: Как бы вы расположили 2, 3, 4, 5, + и =, чтобы составить правильное уравнение?

Ответ: 2 + 5 = 3 + 4

19. Вероятность подбрасывания монеты

Загадка: Если вы подбросили монетку 5 раз и всегда получаете голову. Каковы шансы получить голову, когда бросишь ее в шестой раз?

Ответ: 50-50

20. Число загадок II

Загадка: Как сделать 1 + 9 + 1 = 150 истинным с наименьшими усилиями?

Ответ: Добавьте черту вверху первого +, чтобы получилось 149 + 1 = 150

21.Хорошая загадка для детей

Загадка: Что тяжелее килограмма песка и килограмма ваты?

Ответ: У них одинаковый вес

22. Веселые загадки с числами IV

Загадка: Какое римское число просто CLIX?

Ответ: 159

23. Алгебра

Загадка: Если сложить саму себя, умножить на четыре, а затем разделить на восемь, получится то же число?

Ответ: Весь номер

24.Запись римскими числами

Загадка: Как написать 1, 1000, 51, 6 и 500 римскими числами?

Ответ: IM LIVID

25. Варка яйца

Загадка: Если нужно сварить 4 яйца за 16 минут. Сколько времени нужно, чтобы сварить два яйца?

Ответ: То же 16 минут

26. Игра с числами

Загадка: Какие два целых числа вы получите одинаковый ответ, если умножите их и разделите?

Ответ: Одно и любое другое целое число

27.Двойной рост

Загадка: В озере растет гиацинт, который через день вырастает вдвое. Если сегодня он покрыл все озеро, какой процент озера он покрыл днем ​​ранее?

Ответ: 50 процентов

28. Номера на клавиатуре набора номера смартфона

Загадка: Какое число вы получите, если умножите все числа на циферблате смартфона?

Ответ: Ноль

29.Рост миссис Эльзы

Загадка: Если миссис Эльза на 100 сантиметров на половину своего роста. Какой высоты она?

Ответ: Она 200 сантиметров

30. Делимся апельсинами

Загадка: Как разделить два апельсина с тремя братьями и сестрами, чтобы младший получил один апельсин, не рассердив двух других?

Ответ: Дайте двум старшим братьям и сестрам по апельсину, затем попросите их отдать половину младшему.

Вот и полный список загадок с числами. Я надеюсь, тебе было весело!

Посмотрите это короткое видео:

число загадок с ответами

Подобных загадок для вас:

Лучшие загадки с ответами

Слова загадки

Школьные загадки

Логические загадки

Загадки на Хэллоуин

Рождественские загадки

Библейские загадки

Число загадок

.

Добавить комментарий

Ваш адрес email не будет опубликован. Обязательные поля помечены *